Emergency Med RR Assessment
A 34-year-old man presents to the emergency department complaining of severe eye pain, photophobia, and a foreign body sensation. The symptoms started after the patient was welding two metal pipes together without eye protection. On exam, the clinician appreciates but could not remove the foreign body. Which of the following details of this clinical scenario necessitates evaluation by an ophthalmologist on the day of presentation? AForeign body sensation BPhotophobia CRetained foreign body DSevere eye pain
C Retained foreign body A retained foreign body requires evaluation by an ophthalmologist on the day of presentation. Other findings that require same-day evaluation include a corneal infiltrate, white spot, or opacity that suggests ulceration or pus in the anterior chamber. Patients who need urgent referral, but not same-day evaluation, include a larger epithelial defect, purulent discharge, a drop in vision of more than one to two lines on a Snellen chart, and corneal abrasions that have not healed after three to four days. Infants or children with persistent discharge or unwillingness to keep the eye open should also prompt urgent evaluation or follow-up. These findings suggest a retained foreign body, poor healing, superinfection, or infectious keratitis. Corneal abrasions occur when there is a disruption in the corneal surface epithelium that typically occurs from mechanical trauma to the surface of the eye. Abrasions are usually related to a foreign body, contact lens use, or are spontaneous. Patients commonly present with severe eye pain, photophobia, foreign body sensation, and inability to open the eye. Penetrating trauma must always be ruled out with these symptoms. Infectious infiltrate from herpes simplex virus infection must also be ruled out. Fluorescein staining after completing a complete eye exam will confirm the diagnosis. Trauma to the globe, history of material falling or flying into the eye, history of prior abrasion, awakening in the middle of the night with eye pain, or eye pain on awakening should be treated with erythromycin ointment if possible because of its antibiotic properties and lubricant function. Alternatives to erythromycin are sulfacetamide 10%, polymyxin/trimethoprim, ciprofloxacin, or ofloxacin drops. This treatment should be applied four times per day for three to five days. Pain relief with narcotics is optional but often needed. Pressure patching the eye for less than 24 hours is optional but is contraindicated in the event of a retained foreign body. If the patient wears contact lenses, the patient should be evaluated for an infiltrate or opacity and must be referred to an ophthalmologist as previously described.
At what spinal cord level can an injury cause cessation of respiration?
C3. Injuries at this level are often fatal in the acute setting.
A 40-year-old woman presents after she sustained a painful bite by an unknown insect three days ago. She was cleaning out her attic when she felt a painful bite on her hand. The bite was initially described as a red plaque that has progressed to a dusky red eschar. Physical examination reveals a necrotic ulcer. Which of the following spiders most likely bit her? ABlack widow spider BBrown recluse spider CFunnel web spider DYellow sac spider
B Brown recluse spider Brown recluse spiders belong to the genus Loxosceles and are also known as Loxosceles reclusa. Identification of a brown recluse spider on the basis of body morphology is less reliable than counting the eyes. Brown recluse spiders have six eyes, with a pair in front and a pair on either side with a gap between the pairs. Other spiders have eight eyes. Brown recluse spiders are brown with a violin pattern on the anterior cephalothorax and are distributed in the Central Midwestern United States. They are typically found in dark, quiet, and undisturbed areas, such as basements, attics, behind bookshelves, and under rocks. Clinical manifestations of a brown recluse spider bite or loxoscelism include a painful bite reported by patients that is followed by local signs or systemic effects or both. Local signs may be identified as two puncture marks surrounded by an area of erythema. The bite is usually a painful red plaque or papule. Pain typically intensifies after two to eight hours and may become severe. Over several days, the red plaque or papule develops a dusky red or blue color in the center of the lesion forming an eschar. As the eschar breaks down, a necrotic ulcer typically develops. Systemic effects include malaise, fever, myalgias, nausea, or vomiting. Rarely, systemic effects may include angioedema, acute hemolytic anemia, rhabdomyolysis, myonecrosis, coma, and even death. Diagnosis is made clinically based on the history of feeling a bite and the presence of a necrotic wound. Definitive diagnosis is made only when a patient has a consistent skin lesion that was inflicted by an observed spider that was then recovered and properly identified by an entomologist. Laboratory studies are usually unnecessary initially when only local signs are present but may be needed in the later stages as systemic effects develop. Acute hemolysis, rhabdomyolysis, or acute renal injury may be seen on lab study. Treatment may be achieved by extensive wound cleaning and analgesia with nonsteroidal anti-inflammatory drugs or opioids as needed. Excision/debridement of the wound in the necrotic stage is not recommended but once the wound is stable, this may help expedite wound healing.
A 22-year-old woman with a history of diabetes mellitus presents with abdominal pain and vomiting. She has not been taking her insulin because she has been eating less the past couple days. Her initial serum glucose is 270 mg/dL. If the patient is in diabetic ketoacidosis, which of the following additional findings is most likely? AArterial pH of 7.4 BDecreased serum bicarbonate CHypernatremia DHypokalemia
B Decreased serum bicarbonate Diabetic ketoacidosis is one of the most serious and acute complications of diabetes mellitus. It is characterized by hyperglycemia and ketoacidosis. Diabetic ketoacidosis is more common in younger patients with type 1 diabetes mellitus. Hyperglycemia is caused by impaired glucose utilization, increased gluconeogenesis, and increased glycogenolysis. Gluconeogenesis results from the breakdown of fat and muscles and is promoted by insulin deficiency. Ketoacidosis results from lipolysis and the synthesis of ketones from free fatty acids. Diabetic ketoacidosis is typically triggered by precipitating events, such as infection (pneumonia and urinary tract infections are most common), inadequate insulin, or noncompliance with insulin. It may be the initial presentation of type 1 diabetes mellitus. Diabetic ketoacidosis presents with symptoms related to hyperglycemia, abdominal symptoms, dehydration, and symptoms of the precipitating infection. Hyperglycemic symptoms include polyuria, polydipsia, and weight loss. Abdominal symptoms, such as abdominal pain, nausea, and vomiting, are more common in severe diabetic ketoacidosis, caused by delayed gastric emptying and ileus, and are due to acidemia. Neurologic manifestations, such as decreased awareness and stupor, may be seen in severe diabetic ketoacidosis but are more common in hyperosmolar hyperglycemic state. Hyperventilation with fruity breath is common in diabetic ketoacidosis. Dehydration manifests as tachycardia, orthostatic hypotension, dry mucous membranes, and decreased skin turgor. The diagnosis of diabetic ketoacidosis is confirmed with laboratory testing. The most common laboratory findings of diabetic ketoacidosis are hyperglycemia, hyperosmolality, elevated anion gap, decreased serum bicarbonate, urine ketones, and serum ketonemia. Beta-hydroxybutyrate is often measured to check for elevation in serum ketones. The arterial pH from an arterial blood gas helps to decipher the severity of diabetic ketoacidosis. In general, an arterial pH of 7.25-7.30 indicates mild diabetic ketoacidosis, whereas an arterial pH less than 7.0 indicates severe diabetic ketoacidosis. Patients with diabetic ketoacidosis are typically managed in an intensive care unit. The treatment involves intravenous fluids to help with dehydration and hyperosmolality, potassium, insulin, and sometimes sodium bicarbonate. Patients with diabetic ketoacidosis typically have depleted total body potassium. However, due to transcellular shift, the initial serum potassium may be normal or elevated. The administration of insulin moves potassium back into the cells, which can cause sudden development of hypokalemia. Therefore, patients with an initial potassium between 3.3-5.3 mEq/L should be given intravenous potassium chloride along with intravenous fluids and insulin. Patients with diabetic ketoacidosis and an initial potassium below 3.3 mEq/L should be given intravenous fluids and potassium prior to the administration of insulin. Insulin is given intravenously in moderate-severe diabetic ketoacidosis. An initial bolus may be given, and then a continuous insulin infusion (insulin drip) follows. Sodium bicarbonate is recommended in cases with an arterial pH below 6.9. The serum glucose should be monitored hourly. The ketoacidosis can be monitored every two to four hours with venous blood gas, chemistry panels, or serum ketone measurements.
What are the classic physical exam findings that may be seen in basilar skull fracture?
Battle sign, raccoon eyes, hemotympanum, CSF otorrhea and rhinorrhea.
Which organ do fistulas from diverticulitis most commonly involve?
Bladder
A 42-year-old man reports to the clinic complaining of tenderness and swelling over his elbow. He noticed the area the day after he hit his elbow on a door frame. He has a history of asthma, and he takes fluticasone daily. Examination of the joint reveals an oval-shaped swelling over the olecranon process of the ulna without erythema, warmth, or overlying skin lesions. Which of the following is the best initial treatment for this patient? AAspiration BElbow brace locked in full extension CGlucocorticoid injection DIbuprofen
D Ibuprofen Olecranon bursitis results from inflammation of the bursal sac that overlies the posterior olecranon process of the ulna. This inflammation can occur secondary to trauma, hemorrhage, sepsis, or arthritic conditions, such as gout, rheumatoid arthritis, or osteoarthritis. It commonly causes focal tenderness and an oval swelling at the tip of the elbow. Joint motion is not affected, and extension of the elbow does not accentuate pain. Patients may report tightness or pain with end-range flexion of the joint. Septic bursitis may be indicated with overlying tenderness, erythema, warmth, cellulitis, or skin lesions. Imaging of the area is not indicated in traumatic olecranon bursitis, however, physical exam should evaluate for any signs of complications, such as overlying breaks in the skin or signs of infection. The bursa should be aspirated if infection is suspected or to diagnose a microcrystalline disorder, such as gout. Aspirated fluid should undergo cell count, Gram stain, and culture. The primary treatment for olecranon bursitis is joint protection with elbow pads, sleeves, and orthoses. This treatment, however, should not impede normal range of motion of the joint. Patients with underlying microcrystalline conditions should be treated accordingly with analgesics and appropriate urate lowering therapies, such as colchicine. Infectious cases require prompt initiation of antibiotic therapy (clindamycin, dicloxacillin) and drainage of the infected bursal sac. Therapeutic fluid aspiration may alleviate pressure in patients with aseptic bursitis, but repeated aspiration is not recommended as the fluid will commonly reaccumulate regardless, and the procedure introduces trauma and predisposes the patient to infection. Anti-inflammatory agents (ibuprofen, meloxicam) can be used in those without contraindications. Glucocorticoid injection, while helpful in treating other bursitis syndromes, is not recommended for patients with olecranon bursitis as it can cause skin atrophy and has not been clinically proven to be beneficial. Chronic, persistent olecranon bursitis may require arthroscopic surgical removal of the bursal sac.
Why does testicular torsion cause infertility even with a contralateral normal testis?
Because disruption of the "blood-testis" barrier leads to the formation of anti-sperm antibodies.
Which of the following represents appropriate health maintenance counseling for a patient with hepatic insufficiency and several previous episodes of hepatic encephalopathy? ADaily oral probiotics BLow-protein diet CTwo or three large meals daily with no snacking DZinc supplementation
A Daily oral probiotics Hepatic encephalopathy refers to neurologic changes as the result of hepatic insufficiency. Most patients with hepatic encephalopathy have increased levels of arterial ammonia (which is neurotoxic) due to decreased clearance of ammonia via hepatic enzymes. Other causes of hepatic encephalopathy include electrolyte imbalance, oxidative stress, and elevated 3-nitrotyrosine levels. All patients with hepatic encephalopathy have underlying liver disease (usually severe), and precipitating factors for the development of hepatic encephalopathy include infection, drugs, dehydration, portosystemic shunting, constipation, gastrointestinal bleeding, and excess dietary protein intake. Signs and symptoms of hepatic encephalopathy include flapping of the hands on forced dorsiflexion (asterixis), slurred speech, ataxia, memory deficits, slowed motor reaction time, increased deep tendon reflexes, agitation, depression, and sleep disturbances. Diagnosis of hepatic encephalopathy is with laboratory testing, which will demonstrate hepatic failure and may demonstrate hyperammonemia, as well as computed tomography of the head, which will rule out other neurologic causes of encephalopathy and will demonstrate focal or diffuse cerebral edema. Electroencephalogram can be employed in cases of mild hepatic encephalopathy where the diagnosis is unclear. Treatment of hepatic encephalopathy in the setting of elevated arterial ammonia levels is with oral lactulose, titrated to a dose that leads to two or three soft stools daily. Rifaximin can be administered if lactulose is not well tolerated or not sufficiently efficacious. For patients who suffer repeated episodes of acute hepatic encephalopathy, chronic maintenance administration of lactulose should be prescribed. Maintaining adequate daily nutrition is important for these patients. Daily oral probiotics, such as lactobacillus, help to maintain a healthy gut flora that aids in ammonia elimination.
A 12-year-old boy presents to the emergency department after being hit in the right eye by a baseball. The patient complains of pain with eye movement and diplopia. On physical exam, the right eye is lower in the horizontal plane than the left eye, and enophthalmos is noted on the right. When asked to gaze upward, the left eye responds but the right eye remains stationary. Which of the following physical exam findings best correlates with this patient's probable injury? ADecreased sensation along the right cheek, upper lip, and gingiva BDecreased sensation at the tip of the nose CRight-sided facial droop with muscle weakness including the forehead DWidened intercanthal distance
A Decreased sensation along the right cheek, upper lip, and gingiva Orbital blow-out fracture, also known as an orbital floor fracture, is the result of force applied to the globe that is transmitted to the orbital floor, resulting in either a linear fracture that may return to its original position (more likely in young children), or a shattering of the orbital floor (likely in adults). The mechanism of injury in an orbital floor fracture is usually direct force to the globe by a small round object (such as a ball). The majority of patients presenting with orbital floor fracture are young men. Presenting symptoms of orbital floor fracture include diplopia and pain with eye movement as the result of entrapment of the inferior rectus muscle. Physical exam may reveal a globe that appears sunken in the orbit (enophthalmos) or lower on the horizontal plane that its counterpart (orbital dystopia) as a result of entrapment of the inferior fat pad. Upward gaze may be limited in the injured eye due to entrapment of the inferior rectus muscle. Because the infraorbital nerve passes under the globe, an orbital floor fracture can disrupt sensation to the cheek, upper lip, and gingiva of the injured side. Diagnosis of orbital floor fracture is made with computed tomography. Treatment of orbital floor fracture may involve surgery, especially if there is entrapment of extraocular muscles, facial asymmetry, diplopia, or persistent nausea and vomiting (as a result of a vasovagal response to extraocular muscle entrapment). While the patient awaits surgery, cold packs and oral steroids may decrease swelling, antiemetics should be prescribed for intractable vomiting, and prophylactic antibiotics can be administered to prevent sinusitis in any patients with orbital fractures extending into the sinuses.
A 54-year-old man with a history of smoking and alcohol use disorder reports to the emergency department complaining of six days of abdominal pain. He describes the pain as episodic, epigastric pain that radiates to the back. It is partially relieved by leaning forward and worsens 20 minutes after eating. He also reports nausea and vomiting with loose, greasy, foul-smelling stools. Which of the following laboratory results is most likely abnormal and aids in the diagnosis of this patient? AFecal elastase BFecal fat evaluation with Sudan staining CSerum amylase DSerum lipase
A Fecal elastase Chronic pancreatitis is a fibrotic disorder that most commonly results from alcoholism. Tobacco smoking is an independent risk factor and may accelerate the progression of this condition in patients with alcohol use disorder. Obstructive chronic pancreatitis results from a stricture, stone, or tumor that interferes with the normal exocrine function of the gland. Other causes of chronic pancreatitis include malnutrition, hyperparathyroidism, autoimmunity, genetic predisposition, and recurrent and severe acute pancreatitis. Patients with chronic pancreatitis complain of chronic pain with recurrent episodes of acute pancreatitis and resultant pancreatic exocrine and endocrine insufficiency. Pain is most frequently present in the epigastric and left upper quadrant and anorexia, nausea, vomiting, constipation, flatulence, and weight loss are common presenting complaints. Acute attacks can vary in duration from a few hours to two weeks. Bulky, foul-smelling, fatty stools define steatorrhea and do not often present until 90% of pancreatic function is lost. Laboratory findings may demonstrate a slightly elevated or normal serum amylase and lipase due to the fibrotic destruction of these enzymes within the pancreas. Compression of the bile duct causes elevation of serum alkaline phosphatase and bilirubin. Stool analysis is required to definitively diagnose the condition with demonstration of pancreatic insufficiency. This is best accomplished using fecal elastase testing, which is decreased in these patients. A 72-hour fecal fat quantitative analysis is the gold standard diagnostic test but is not as practical given the duration of the test and the time taken to analyze results. A secretin stimulation test can be performed to quantify the normal pancreatic secretory content. Noninvasive diagnostic testing includes the use of CT scan, magnetic resonance imaging, or magnetic resonance cholangiopancreatography (MRCP). Diagnosis can be established with evidence of calcification within the pancreas on CT scan or abnormal pancreatogram results. Endoscopic ultrasonography is an invasive diagnostic study that requires a highly skilled gastroenterologist and is, therefore, not as prevalent as the noninvasive methods of diagnosis. Endoscopic retrograde cholangiopancreatography is the most sensitive imaging study for chronic pancreatitis and affords clinicians the opportunity to intervene during diagnostic imaging. Patients with chronic pancreatitis are highly likely to develop type 2 diabetes mellitus within 25 years of the clinical onset of their pancreatic disease. A low-fat diet with avoidance of alcohol should be advised in all patients. Agents for pain control include acetaminophen, nonsteroidal anti-inflammatory drugs, tramadol, and pain-modifying agents such as tricyclic antidepressants (amitriptyline), selective serotonin reuptake inhibitors (fluoxetine), and gabapentin or pregabalin. Opioid pain medications should be avoided due to their addictive potential and the long-term use of pain medications in patients with chronic pancreatitis. Pancreatic insufficiency can be treated with oral pancreatic enzyme supplementation with meals. Concurrent administration of an H2 receptor blocker (ranitidine) or a proton pump inhibitor (omeprazole) can decrease the inactivation of the enzyme by gastric acid. Autoimmune cases of chronic pancreatitis can be treated with prednisone oral therapy for one to two months with a subsequent taper. Refractory patients can be treated with rituximab for induction of remission, and azathioprine or low-dose long-term corticosteroid regimens are used for maintenance therapy. Endoscopic intervention and surgery are indicated in patients refractory to medical approaches.
A 54-year-old man with prior history of myocardial infarction presents to the emergency department complaining of severe left eye pain. Pain has been intermittent but will last for 15-20 minutes when it occurs. He reports the pain is so severe that it will make his left eye water and become very red. He has not been able to get relief from over-the-counter medications. He is examined during an attack of symptoms, which reveals left eye miosis, conjunctival injection, and lacrimation. He is visibly uncomfortable and rocking back and forth on the examination table. Based on the likely diagnosis, which of the following therapeutic interventions would be performed in the emergency department for abortive relief? AHigh-flow oxygen BIntranasal lidocaine COral zolmitriptan DSubcutaneous sumatriptan
A High-flow oxygen Cluster headache is a trigeminal autonomic cephalgia resulting from the activation of the trigeminal-hypothalamic pathway. High-flow oxygen is the treatment of choice for cluster headache. Oxygen is first line due to its efficacy, safety, and lack of side effects. However, in the outpatient setting it is difficult to obtain and expensive. Cluster headache more commonly affects men. Tobacco use and family history of cluster headaches are associated with an increased risk of development. Patients often complain of severe unilateral orbital or supraorbital pain with autonomic features, restlessness, and agitation. Autonomic features can include unilateral tearing, conjunctival injection, rhinorrhea, miosis, and ptosis. The headache attacks can last for 15-180 minutes and can occur multiple times per day. Clusters can last for weeks at a time. Magnetic resonance imaging of the brain is initially recommended at the time of diagnosis to rule out an underlying brain lesion. High-flow oxygen through a nonrebreathing mask is first-line treatment. It is used for at least 15 minutes. Other options include intranasal triptans, lidocaine, and subcutaneous sumatriptan, provided there are no contraindications to triptan usage. Preventative therapies should be started at the onset of a cluster episode to suppress further attacks and reduce the duration of headaches. Verapamil is the treatment of choice for initial preventive therapy. Topiramate and lithium are also possible options if verapamil is not tolerated or not effective. Prednisone can be used in the interim to bridge to preventative therapy.
A 10-year-old boy presents to the emergency department complaining of right eye redness, pain, photophobia, and decreased visual acuity after being struck in the face with a baseball. Physical exam reveals blood pooling in the inferior portion of the anterior chamber, obscuring the inferior one-third of the iris and inferior portion of the pupil. The visible portion of the pupil is round and reactive to light. Swinging light test is negative for afferent pupillary defect on the right. Extraocular muscle range of motion is normal bilaterally. X-ray of the skull is negative for orbital fracture. Fluorescein staining is negative for corneal abrasion. Which of the following is the most likely diagnosis? A Hyphema B Open-globe rupture C Subconjunctival hemorrhage D Traumatic iritis
A Hyphema Hyphema refers to gross pooling of blood in the anterior chamber of the eye. The most common cause of hyphema is traumatic eye injury, generally sports-related in younger children and more likely to be assault in adolescents and adults. The symptoms of hyphema include eye pain, redness, decreased visual acuity, photophobia, nausea, vomiting, and drowsiness. The sign of a hyphema is pooling of blood in the anterior chamber, although these injuries may also be accompanied by decreased visual acuity, anisocoria, corneal abrasion, open-globe rupture, elevated intraocular pressure, lens subluxation, and bradycardia. Open-globe rupture must be ruled out before pressure is applied to the eye during physical exam and before drops are instilled. A patient presenting with a hyphema and no history of ocular trauma should be tested for diabetes mellitus, clotting disorders, juvenile xanthogranuloma, and eye tumors. Medications that increase the risk of bleeding, such as warfarin and aspirin, can also cause nontraumatic hyphema. Treatment of hyphema requires ophthalmologic consultation, and preservation of vision is often dependent on time to treatment and hyphema grade. Blood which pools under one-third of the anterior chamber is classified as a grade I hyphema. One-third to one-half of the anterior chamber covered by pooled blood is classified as grade II hyphema. Over 50 percent but under 100 percent filling of the anterior chamber with blood is grade III, and 100 percent is grade IV. Those with grade I hyphema have a 90% rate of recovery of 20/50 vision or better with prompt management of the hyphema. Those with grade III or IV hyphema have a 50% chance of recovery of 20/50 vision. While ophthalmologic consultation is pending, the patient should be put in a quiet, dimly lit room and rest on a bed with 30 degrees of head elevation to facilitate hyphema resolution. The patient should rest and not read or exert the injured eye. A protective eye shield should be placed, antiemetics administered, and coagulopathies addressed. Analgesic eye drops and cycloplegic eye drops can be administered for pain control only after open-globe rupture has been ruled out via computed tomography.
A 22-year-old man presents to the emergency department after injuring his dominant right shoulder while playing hockey. He describes tripping and falling forward with his right arm up over his head in abduction and external rotation. X-rays show an anterior dislocation of the glenohumeral joint. The shoulder is reduced and postreduction X-rays show anatomic reduction and no fractures. Which of the following anatomic lesions is most likely to be present? AAvulsion of the glenohumeral ligament from the humerus BAvulsion of the glenoid labrum from the anterior inferior glenoid CSubscapularis tendon tear DSuperior labral anterior-posterior tear
B Avulsion of the glenoid labrum from the anterior inferior glenoid Dislocations of the shoulder involve the glenohumeral joint. The majority are anterior dislocations with the humeral head displaced anteriorly relative to the glenoid. Posterior and inferior dislocations can also occur. The glenoid is shallow, which allows for a large range of motion. The capsular ligaments, especially the inferior glenohumeral ligament, and glenoid labrum provide static stabilization, and the rotator cuff tendons provide dynamic stabilization. The usual mechanism of anterior shoulder dislocation involves a force on the hand or forearm while the arm is in a position of forward elevation, abduction, and external rotation, levering the humeral head out of the glenoid. This dislocation results in injury to the tissues. A Bankart lesion is an avulsion of the glenoid labrum from the inferior glenoid. It may involve avulsion of a small fragment of bone ("bony Bankart lesion"), but in most patients under 30 years old, it involves the soft tissues alone. Physical examination findings include loss of the normal contour of the shoulder with a "squared off" appearance. The acromion may appear prominent, and the arm is positioned in slight abduction and external rotation. A neurovascular examination should be done before and after reduction since neurapraxia of the axillary nerve can occur. There may be an associated fracture, so X-rays should also be performed before and after reduction. Multiple reduction maneuvers have been described that typically involve a combination of traction on the arm and scapular manipulation. The most common complication of anterior shoulder dislocation in patients under 30 years old is recurrent dislocation. Immobilization of the shoulder after reduction has not been definitively shown to decrease the risk of recurrence. Rehabilitation is the mainstay of treatment.
A 45-year-old man with diabetes mellitus presents with burning pain of the glans penis for four days. Physical examination shows a curd-like exudate and small papules with blotchy erythema and glazed appearance. Which of the following is the most appropriate therapy? A Cephalexin B Clotrimazole C Fluconazole D Metronidazole
B Clotrimazole Balanitis is characterized by inflammation of the glans penis. When the prepuce in an uncircumcised male also becomes infected, the inflammation is then referred to as balanoposthitis. Balanitis is more common in men of African or Hispanic descent and is predominantly associated with inadequate hygiene in uncircumcised men. Predisposing factors include diabetes mellitus, trauma, obesity, and edematous conditions (such as cirrhosis, congestive heart failure, and nephrotic syndrome). Infectious agents (such as candida, and bacteria), dermatologic disorders (psoriasis, lichen planus, eczema), and premalignant conditions (Bowenoid papulosis, erythroplasia of Queyrat) have been associated with balanitis. Patients present with burning pain, tenderness, or pruritus of the glans penis and the prepuce that has evolved over the previous three to seven days. Physical examination may reveal erythema associated with a curd-like (candida) or foul-smelling purulent (anaerobic bacterial) exudate and ulceration or circinate balanitis (associated with reactive arthritis). Complications of balanitis include phimosis, paraphimosis, reactive arthritis, and premalignant conditions. Diagnosis is typically made clinically, but additional evaluation may be warranted including bacterial and fungal culture, herpes simplex virus testing, syphilis testing, scabies testing, and trichomonas testing. Treatment of balanitis without an identifiable cause involves hygiene measures and empiric anticandidal treatment using antifungal agents (clotrimazole 1%, miconazole 2%). For severe cases, oral fluconazole or a combination of topical imidazole and hydrocortisone 1% cream is typically indicated. When bacterial balanitis is suspected, empiric topical metronidazole 0.75% should be initiated. In the setting of severe bacterial disease, systemic metronidazole, cephalexin, amoxicillin-clavulanate, or clindamycin may be used.
A 46-year-old man presents to the clinic for a routine physical. He has a medical history of coronary artery disease, hypertension, hyperlipidemia, and a previous myocardial infarction. His vitals reveal HR 75 bpm, RR 15/min, BP 148/87 mm Hg, T 98.7°F, and SpO2 99% on room air. His most recent lipid panel reveals low-density lipoprotein cholesterol of 92 mg/dL, high-density lipoprotein cholesterol of 41 mg/dL, triglyceride level of 394 mg/dL, and total cholesterol of 212 mmol/L. Which of the following should be the primary goal in reducing this patient's risk for acute coronary syndrome? A Decreasing blood pressure using amlodipine B Decreasing low-density lipoprotein levels using atorvastatin C Decreasing triglyceride levels using gemfibrozil D Increasing high-density lipoprotein levels using niacin
B Decreasing low-density lipoprotein levels using atorvastatin Coronary heart disease is a pathologic process that affects the coronary circulation and results in acute coronary events, such as angina, myocardial infarction, and cerebrovascular accident. It is the number one cause of death worldwide and is responsible for approximately one in five deaths in the United States each year. Risk factors for developing coronary artery disease include family history of premature coronary heart disease, smoking, hypertension, high-density lipoprotein levels below 40 mg/dL, and age ≥ 45 in men or ≥ 55 in women. Additionally, patients with diabetes mellitus and those who consume too much alcohol and too few fruits and vegetables are at an increased risk for developing coronary heart disease. High-density lipoprotein levels ≥ 60 mg/dL have been demonstrated to be cardioprotective. Smoking remains the most important modifiable risk factor, with the risk of coronary heart disease decreasing by 50% one year after smoking cessation. Risk factor reduction is an important component of reducing the morbidity and mortality in susceptible patients. In clinical trials, only the reduction of elevated low-density lipoprotein cholesterol levels has demonstrated clinical benefit. Medical management of elevated serum triglycerides or low levels of high-density lipoprotein cholesterol has not shown significant benefit in the primary prevention of coronary heart disease. Studies have demonstrated that hydroxymethylglutaryl-coenzyme A (HMG-CoA) reductase inhibitors (also called statins) are effective in preventing death, coronary events, and stroke and should be administered to all patients at risk of these events regardless of cholesterol levels. Although an overall target low-density lipoprotein cholesterol level is not recommended, many guidelines for statin therapy recommend that those with prior cardiovascular events maintain low-density lipoprotein cholesterol levels below 70 mg/dL. High-intensity statins include atorvastatin and rosuvastatin in doses of 40-80 mg or 20-40 mg, respectively. This group tends to lower low-density lipoprotein cholesterol levels by over 50%. Low-intensity statin therapy lowers low-density lipoprotein levels by less than 30% and consists of low-dose simvastatin, pravastatin, lovastatin, fluvastatin, and pitavastatin. Moderate-intensity statin therapy includes those agents used for high-intensity therapy at lower doses as well as higher doses of low-intensity medications. Patients at risk for coronary heart disease benefit most from high- or moderate-intensity statin therapy. Myalgia is the primary complaint that causes statin intolerance and can be exacerbated in patients with hypothyroidism. Other side effects include elevated aminotransferases, proteinuria, increased risk of diabetes mellitus, and drug-induced lupus. Statins are a pregnancy category X drug and are contraindicated in pregnant and nursing women.
A 65-year-old man with chronic low back pain presents to the emergency department with burning pain in his lower back that has progressively worsened over the last three days. He reports that the pain radiates down the posterior aspect of his legs bilaterally below the knee. Physical examination reveals positive straight leg raise and diminished patellar reflex. Upper extremity strength is 5/5. Lower extremity strength is 2/5 bilaterally. Magnetic resonance imaging reveals an unequivocal disc herniation in the L3-L4 nerve roots. Which of the following is the most appropriate therapy? A Acetaminophen B Discectomy with limited laminotomy C Epidural corticosteroids D Gabapentin
B Discectomy with limited laminotomy The patient in the vignette above has sciatica due to L3-L4 disc herniation. Sciatica is pain along the sciatic nerve typically due to nerve root compression in the lower back that is commonly seen in the L3-L4, L4-L5, or L5-S1 nerve roots. Causes of sciatica include intervertebral disk herniation, bony irregularities (including osteoarthritis, osteophytes, and spondylolisthesis), spinal stenosis, and intraspinal tumor or abscess. Patients with sciatica typically present with a burning, lancinating, or stabbing pain that radiates down the buttocks and posterior aspect of the leg below the knee. Pain may be unilateral or bilateral and is commonly aggravated by Valsalva or coughing. Weakness and numbness may also be present in the affected leg. Physical examination may reveal motor, sensory, or reflex deficits. L3-L4 disc herniation typically affects the patellar reflex, while the L5-S1 herniation affects the ankle jerk reflex. Straight leg raise above 60° may cause pain to radiate to the posterior aspect of the affected leg. Crossed straight leg raise test may be positive. Pain radiation with the slump test (slumping with the thoracic and lumbar spines flexed) may also be present. Initial diagnosis of sciatica is clinical but is confirmed by magnetic resonance imaging (MRI) or electrodiagnostic studies if symptoms persist for more than six weeks or if there are motor, sensory, or reflex deficits. MRI may show structural abnormalities, such as spinal stenosis. Electrodiagnostic studies may exclude other conditions that mimic sciatica, such as polyneuropathy. Treatment of sciatica involves conservative acute pain management using nonopioid analgesics (such as acetaminophen), heat therapy, cold therapy, and physical therapy. Gabapentin and epidural corticosteroids may provide some pain relief. Surgical intervention (classic discectomy with limited laminotomy) is only indicated for cauda equina syndrome or unequivocal disc herniation and one of the following: muscular weakness, progressive neurological deficits, and intractable pain that interferes with job or personal life in an emotionally stable patient who has failed conservative treatment.
A 22-year-old man presents to the ED after falling while skateboarding. The patient was initially alert and talking to you, but is becoming progressively more somnolent. A non-contrast computed tomography scan of his head is shown above. Which of the following pairs represents the correct diagnosis and most likely injured vessel? A Epidural hematoma, bridging veins B Epidural hematoma, middle meningeal artery C Subdural hematoma, bridging veins D Subdural hematoma, middle meningeal artery
B Epidural hematoma, middle meningeal artery This patient's imaging and physical examination are consistent with a traumatic acute epidural hematoma. An epidural hematoma is a collection of blood between the skull and the dura. Epidural hematomas are usually associated with skull fractures in the temporal bone region resulting in laceration of the middle meningeal artery. Arterial bleeding is the etiology of ⅔ of epidural hematomas and is typically rapid. Epidural hematomas are uncommon in the elderly and in children < 2 years of age due to the close attachment of the skull to the dura. Epidural hematomas account for only 1% of all head-injured patients presenting with coma and are present in 0.5% of all head-injured patients. Signs and symptoms include severe headache, drowsiness, nausea, and vomiting. The classic finding is the lucid interval just prior to rapid deterioration; however, this is present in < 30% of epidural bleeds. Diagnosis is via non-contrast CT of the head. This characteristically shows a hyperdense lenticular-shaped hematoma in the temporal region. These hematomas are sharply defined and do not cross suture lines. Management of epidural hematomas is primarily surgical. Neurosurgery should be consulted immediately for surgical evacuation of the hematoma. Prognosis is good if the epidural hematoma is promptly treated and the patient does not present with coma.
A 42-year-old man presents to the clinic complaining of a painful, swollen elbow joint. He states his symptoms were present when he awoke this morning. His past medical history includes diabetes mellitus and hypertension, for which he takes metformin and hydrochlorothiazide. Physical exam findings reveal fever and a tender, warm, erythematous elbow joint. Laboratory analysis of aspirated synovial joint fluid reveals monosodium urate crystals and an elevated white blood cell count. Which of the following diagnoses are consistent with this result? ABasic calcium phosphate crystal disease BGout CPseudogout DSeptic arthritis
B Gout Gout is a disease caused by the dysregulation of uric acid metabolism. Uric acid levels greater than 6.8 mg/dL cause urate to fall out of solution and deposit within tissues and synovial joint fluids. Hyperuricemia can be caused by overproduction or underexcretion of uric acid, or a combination of both. Patients of Pacific Island descent are more commonly affected with the condition. Primary gout expresses recognizable genetic inheritance patterns while secondary gout is linked to acquired causes of hyperuricemia. Medications, such as diuretics, low-dose aspirin, cyclosporine, and niacin, commonly cause hyperuricemic states. Other acquired causes of gout include myeloproliferative disorders, plasma cell myeloma, hemoglobinopathies, chronic kidney disease, hypothyroidism, psoriasis, sarcoidosis, and lead poisoning. Alcohol has been shown to increase urate production and decrease the renal excretion of uric acid and excess consumption can precipitate acute gout flares. Men are more likely to present with primary gout, with increasing incidence after age 30. Tophi are nodular deposits of monosodium urate monohydrate crystals with an associated foreign body reaction that are primarily found in cartilage, subcutaneous and periarticular tissues, tendon, bone, and the kidneys. These deposits most commonly present in the external ears, feet, olecranon and prepatellar bursae, and hands. Acute gouty arthritis has a sudden, frequently nocturnal onset and presents with swollen, tender joints with tense, warm, and dusky red skin overlying the joint. The most commonly affected joint is the metatarsophalangeal joint of the great toe, a condition termed podagra. Fever is frequently present with local desquamation and pruritus appearing during recovery from acute gouty attacks. Patients may be asymptomatic for months or years between attacks. Gout can develop into a chronic, deforming polyarthritis of upper and lower extremities that mimics rheumatoid arthritis. Serum uric acid levels may be elevated or normal during acute flares while leukocytosis is frequently present. Aspiration of synovial joint fluid demonstrates needle-like sodium urate crystals that are negatively birefringent when examined by polarized light microscopy. Plain film radiography reveals no changes in early disease but demonstrates punched-out erosions with an overhanging rim of cortical bone ("rat bite" erosions) later in the disease. Smaller deposits of urate crystals can be found with ultrasonography. Treatment is not indicated in asymptomatic uricemia. Acute attacks should be treated with oral nonsteroidal anti-inflammatory drugs, such as naproxen or indomethacin, unless contraindicated. Colchicine can be administered within 36 hours of acute gout attack onset but should not be repeated within 14 days of the last administration. Corticosteroids (e.g., methylprednisolone, prednisone) provide significant symptom relief and are used in patients with contraindications to nonsteroidal anti-inflammatory drugs. Anakinra, canakinumab, and rilonacept are also effective medications for the management of acute gout but have not been approved by the U.S. Food and Drug Administration for this indication. Between attacks, management goals should focus on reducing circulating urate to prevent further attacks and disease progression. Patients should be instructed on dietary modifications, and hyperuricemic medications should be avoided. Colchicine may be used for prophylaxis while reduction of serum uric acid is accomplished with the use of xanthine oxidase inhibitors (e.g., allopurinol, febuxostat), uricosuric drugs (e.g., probenecid, lesinurad), or pegloticase. Untreated acute gout attacks can last for a few days up to several weeks. As the disease progresses, asymptomatic intervals between attacks shorten.
A 25-year-old man presents to the emergency department with recurrent epistaxis, gum bleeding, and easy bruising. Physical examination is normal. Laboratory studies reveal normal platelet, normal prothrombin, abnormal activated partial prothrombin time, and prolonged bleeding time. Based on the most likely diagnosis, which of the following measures should be undertaken to reduce bleeding episodes? AFactor replacement therapy for two days prior to simple laceration repair BHematology referral prior to dental surgery CParticipation in contact sports DUse of aspirin for pain control
B Hematology referral prior to dental surgery Von Willebrand disease is a coagulation disorder characterized by a quantitative reduction or dysfunction of the von Willebrand factor, a large multimeric glycoprotein that is released at the site of a vascular injury aiding in the formation of the platelet plug. Von Willebrand factor also prolongs the half-life of factor VIII in circulation. Von Willebrand disease is the most common inherited bleeding disorder. There are three types of von Willebrand disease: type 1 (due to a quantitative reduction in von Willebrand factor, most common), type 2 (dysfunctional von Willebrand factor), and type 3 (absent or severely reduced von Willebrand factor). Subtypes of type 2 von Willebrand disease include type 2A (loss of platelet binding of von Willebrand factor), type 2B (gain-of-function in von Willebrand factor leading to enhanced platelet function), type 2M (reduced binding of von Willebrand factor to glycoprotein 1b), and type 2N (low factor VIII). Clinical manifestations of von Willebrand disease depend on the type of von Willebrand disease present and may include mild-to-severe mucosal hemorrhage or bleeding, easy bruising, epistaxis, heavy menstrual bleeding, postpartum bleeding, and gum bleeding with teeth brushing. Hemarthrosis may be seen in the rare types 2N and 3 and is due to decreased levels of factor VIII. The majority of individuals with von Willebrand disease have a normal complete blood count, normal prothrombin time, and activated partial thromboplastin time. However, in individuals with concurrent reduced factor VIII, activated partial thromboplastin time is prolonged. Thrombocytopenia may be seen in the setting of type 2B von Willebrand disease. Microcytic anemia may result due to blood loss from excessive uncontrolled bleeding. Bleeding time is no longer used because it is time-consuming, operator-dependent, and does not correlate with bleeding risk but may be prolonged if used especially with aspirin challenge. Bleeding time may also be normal but does not rule out the possibility of von Willebrand disease. Confirmation of von Willebrand disease is made by von Willebrand screening tests, including von Willebrand activity, antigen, and factor VIII assay. Treatment of mild von Willebrand disease types 1 and 2 is intravenous or intranasal desmopressin. In the setting of continued bleeding, von Willebrand factor plus factor VIII may be given. To help prevent episodes of bleeding, individuals with von Willebrand disease should be encouraged to limit the use of blood thinning medications, such as naproxen, aspirin, and ibuprofen. They should also be advised to see their hematologist before any surgery, dental procedures, childbirth, or other procedures. It is recommended that they abstain from any activities that may cause bruising/bleeding, such as contact sports (football, hockey, soccer, wrestling). A medical bracelet may be appropriate, which can alert medical personnel if the patient is unresponsive.
A 28-year-old man presents to the emergency department with progressive gait dysfunction and leg weakness worsening over the five days. Prior to the development of symptoms, he suffered from a gastrointestinal illness. He reports a family history of multiple sclerosis in his maternal uncle. Physical examination demonstrates symmetric lower extremity muscle weakness, gait ataxia, and absent deep tendon reflexes. Which of the following diagnostic studies would be performed initially to confirm the likely diagnosis? A Antibody testing for GQ1b B Lumbar puncture, evaluating protein level C Magnetic resonance imaging of the brain with gadolinium D Magnetic resonance imaging of the lumbar spine
B Lumbar puncture, evaluating protein level Guillain-Barré syndrome is an immune-mediated polyneuropathy affecting peripheral nerves. There are multiple variants depending on the nerves involved (sensory, motor) and whether it is primarily demyelinating or axonal nerve damage. Lumbar puncture to evaluate the protein level is performed early on in patients with suspected Guillain-Barré syndrome. Risk factors for its development include recent infection (particularly Campylobacter jejuni), recent immunization (influenza vaccination), surgery, bone marrow transplantation, and trauma. Patients present with progressive ascending weakness and difficulty walking. Symptoms are symmetric, beginning in the legs and spreading caudally. Patients can develop paralysis, autonomic dysfunction, bowel and bladder dysfunction, and respiratory failure secondary to weakness of the diaphragm. Neurological examination demonstrates decreased-to-absent reflexes, impaired strength, and gait dysfunction. Lumbar puncture classically reveals elevated protein levels with normal white blood cell counts, however, if early in the disease state the protein level can be normal. Electrodiagnostic studies (electromyography and nerve conduction testing) are also necessary for the diagnosis. In the more common demyelinating forms of Guillain-Barré, nerve conduction studies commonly demonstrate decreased motor nerve conduction velocity and conduction blocks. Magnetic resonance imaging of the lumbar spine may demonstrate thickening and enhancement of intrathecal spinal nerve roots and cauda equina. Intravenous immunoglobulin (IVIG) and plasmapheresis are used in the treatment of Guillain-Barré. Close monitoring is imperative to evaluate for additional interventions such as cardiac monitoring and mechanical ventilation in the event of respiratory failure. Vital capacity and negative inspiratory force (NIF) should be evaluated every four hours. If the negative inspiratory force decreases near 20 cm water, intubation is indicated. Additionally, other factors that indicate a patient may need additional respiratory support include forced vital capacity less than 20 mL/kg, maximum inspiratory pressure less than 30 cm water, or maximum expiratory pressure less than 40 cm water. Patients will need deep venous prophylaxis, physical therapy, and occupational therapy. Patients who are elderly, require mechanical ventilation, or who have more severe symptoms initially have a poorer prognosis.
A 14-year-old boy presents to a rural urgent care clinic with limited resources complaining of sudden-onset scrotal pain associated with nausea and vomiting for three hours. The closest hospital is four hours away. Physical examination reveals bell-clapper deformity. Cremasteric reflex and Prehn sign are negative. Which of the following is the next best step? AAntibiotic therapy BManual detorsion CPain medication DTransfer to the nearest hospital
B Manual detorsion Testicular torsion is acute scrotal pain that arises from the inadequate fixation of the lower pole of the testis to the tunica vaginalis, leading to testicular twisting on the spermatic cord. This twisting may potentially produce ischemia from reduced arterial inflow and venous outflow obstruction. Testicular torsion is considered a urologic emergency as it may lead to irreversible damage and infertility after 12 hours of ischemia. Testicular torsion most commonly affects neonates and postpubertal boys and may be precipitated by an inciting event (e.g., trauma) or spontaneously. Patients typically present several hours after vigorous physical activity or minor testicular trauma with moderate to severe testicular pain, testicular swelling, nausea, and vomiting. Alternatively, children may awake with testicular pain in the middle of the night or in the morning. Physical examination may reveal negative cremasteric reflex, negative Prehn sign, asymmetric high-riding testis (bell-clapper deformity), and pain relief with manual detorsion. Diagnosis may be made presumptively by history and physical examination but color Doppler ultrasound confirms the diagnosis. Management of testicular torsion is urgent surgical consultation for detorsion and fixation. Delay of surgical management (> six hours) may result in infarction of the testis with liquefaction requiring orchiectomy. If surgery is not available within two hours of presentation, manual detorsion should be attempted. Manual detorsion involves using the right thumb and forefinger to twist the affected testicle medial to lateral (clockwise) 180 degrees. Once performed, doppler color ultrasound should be performed to ensure return of adequate blood flow to the testicle. Pain medication should not be administered as it may mask the pain from the torsion. Successful manual detorsion is suggested by the following: relief of pain, conversion of transverse lie of the testis to a longitudinal orientation, lower position of the testis in the scrotum, and return of normal arterial pulsations on color Doppler ultrasound. Orchiopexy should be performed after successful manual detorsion to prevent recurrence and to relieve residual torsion.
A 70-year-old man presents to the emergency department with constant left lower abdominal pain for three days. He says he has been mildly constipated but denies all other symptoms. On abdominal exam, he is tender to palpation in the left lower quadrant. What is the best diagnostic test for the most likely diagnosis? A Abdominal radiograph B Colonoscopy C CT abdomen with intravenous contrast D MRI abdomen
C CT abdomen with intravenous contrast Acute diverticulitis is defined by inflammation of a diverticulum (sac-like protrusions of the colon wall) that is typically due to microperforation. The mean age of diagnosis of acute diverticulitis is 63 years of age, however, 20% of cases occur in patients less than 45 years of age. The classic symptom of acute diverticulitis is a constant left lower quadrant abdominal pain. Other findings may include low-grade fever, nausea, vomiting, constipation, or diarrhea. The abdominal exam findings vary based on the degree of inflammation, ranging from left lower quadrant abdominal tenderness to peritoneal signs, including rigidity and rebound tenderness. Complications of acute diverticulitis include abscess formation, fistula formation, perforation with or without peritonitis, and bowel obstruction (usually partial). Leukocytosis can support the diagnosis but lacks high sensitivity or specificity. The urinalysis may show pyuria due to surrounding inflammation. The best diagnostic study is a CT scan of the abdomen with intravenous contrast. The presence of colonic diverticula, pericolonic fat stranding, and localized bowel wall thickening (more than 4 mm) support the diagnosis of acute diverticulitis. Some patients with acute diverticulitis require hospitalization. These include patients with complicated diverticulitis and uncomplicated diverticulitis with one or more of the following features: sepsis, microperforation, immunosuppression, high fever (> 102.5° F), severe abdominal pain, advanced age, significant comorbidities, intolerance of oral intake, and failing outpatient therapy. Immunosuppression includes use of immunosuppressive agents, chronic high-dose use of corticosteroids, advanced HIV infection, and poorly controlled diabetes mellitus. The outpatient management consists of oral antibiotics for 7 to 10 days, and patients should be reassessed after two or three days of treatment. The outpatient regimen used to treat acute diverticulitis is typically one of the following: ciprofloxacin and metronidazole, levofloxacin and metronidazole, or amoxicillin-clavulanic acid. There is no evidence for dietary restriction in patients with uncomplicated diverticulitis. However, some clinicians recommend a clear liquid diet for two to three days prior to reassessment. The inpatient management for uncomplicated diverticulitis consists of intravenous antibiotics, intravenous fluids, and pain management. Patients can be made nil per os or be offered a clear liquid diet. The recommended antibiotic regimen for patients who are admitted is either piperacillin-tazobactam or one of the following combinations: ciprofloxacin and metronidazole, levofloxacin and metronidazole, or ceftriaxone and metronidazole. Patients with complicated acute diverticulitis are admitted, and the complications are usually treated surgically. Patients with acute diverticulitis should undergo a colonoscopy six to eight weeks following successful treatment. Patients who develop recurrent diverticulitis may require elective surgical bowel resection once the acute inflammation has resolved.
A 28-year-old woman is being evaluated for a self-reported change in mood. Which of the following mood episodes must have occurred to diagnose her with bipolar I disorder? AHypomanic episode BMajor depressive episode CManic episode DPsychotic episode
C Manic episode Patients with bipolar I disorder must have a history of one or more manic episodes. While patients with bipolar I disorder often have periods of depression, even meeting Diagnostic and Statistical Manual of Mental Disorders, volume 5 (DSM-V) criteria for a major depressive episode, this symptom is not a requirement for diagnosis. Manic episodes are characterized by an escalation in mood, which is typically described as feelings of elation, expansiveness, or extreme irritability. Patients often report a decreased need for sleep and may be more socially outgoing during manic periods. The disinhibition observed during mania can manifest as hypersexual behavior, excessive spending, flamboyant outfits, risky business investments, or project initiation. The impaired judgment associated with this behavior can result in painful consequences in the form of strained relationships, financial hardship, or legal troubles. While psychotic features, such as hallucinations and delusions, may be present, they are not required for the diagnosis of bipolar I disorder. Racing thoughts and flight of ideas paired with pressured speech can make the patient's interview challenging. Typical disease course is remitting and relapsing with a significant increase in suicide compared to major depressive disorder. The average age of diagnosis is 30 years old and is often made after patients have been diagnosed with major depressive disorder or anxiety before the true diagnosis is made. There is a strong genetic predisposition, especially in first-degree relatives. Treatment is based on using mood stabilizers, such as lithium, valproic acid, olanzapine, and carbamazepine. Additional therapeutics include gabapentin, topiramate, lamotrigine, and second-generation atypical antipsychotics, which may be used to support the effects of mood stabilizers.
A 69-year-old woman presents to the emergency department with a headache that is worse in the early morning for two days. She has no significant medical history. She reports a pulsatile machine-like sound in her ears. Bilateral funduscopic exam is performed, and retinal findings are shown above. Visual field and visual acuity testing are normal. Which of the following is the most likely diagnosis? AAnterior ischemic optic neuropathy BCentral retinal vein occlusion CPapilledema DPapillitis
C Papilledema Papilledema is a term used to describe optic disc swelling due to increased intracranial pressure. The increase in pressure causes axonomic flow obstruction within the nerve, resulting in axon swelling and leakage of water, proteins, and other cellular contents into the extracellular space of the optic disc giving rise to optic disc swelling. Conditions that cause increased intracranial pressure, which may lead to papilledema, include intracranial mass lesions, cerebral edema, increased cerebrospinal fluid, pseudotumor cerebri, and obstructive hydrocephalus. Patients with papilledema usually present with bilateral optic disc swelling, a headache that is worse in the early morning, and binocular horizontal diplopia. Patients may also report a pulsatile machine-like sound in the ears. Visual symptoms are a rare finding in acute papilledema. However, untreated chronic papilledema may lead to progressive vision loss and even blindness. Fundoscopic findings in papilledema may evolve over time and are classified as early (spontaneous venous pulsation), fully developed (optic disc elevation, cup obliteration, and disc margin blurring), and chronic or late (hard exudates, flattened nerve, and disc pallor). Neuroimaging using computed tomography or magnetic resonance imaging is promptly indicated if papilledema is present on fundoscopy. If neuroimaging is normal, lumbar puncture is warranted. Other tests that may be done in the setting of papilledema include visual field testing, fluorescein angiography, and optical coherence tomography. Treatment of papilledema involves treating the underlying etiology.
A 12-year-old girl presents to the emergency department after injuring her left knee earlier today. She is a gymnast and landed awkwardly from a vault. She states that her knee "collapsed" when she landed, and she heard and felt a "pop" in the knee as she fell. The knee was very painful, and she was unable to extend it or put weight on the leg. An athletic trainer who witnessed the injury had the patient lay on her back on the mat and was able to bring her knee into extension. As he did so, the patient felt another "pop" in the knee and immediately had less pain. She was helped off the mat but still had difficulty putting weight on the knee. On physical examination, she has a large effusion in the knee. Range of motion of the knee is 0° of extension compared with +15° of hyperextension on the right and 120° of flexion compared with 150° on the right. She has diffuse tenderness to palpation over the knee and a positive apprehension sign. She has no laxity to varus or valgus stress, Lachman, or drawer testing but is guarding. X-rays of the knee show a joint effusion but no bony abnormalities. Her physes are open but beginning to fuse. Which of the following is the most likely diagnosis? AAnterior cruciate ligament tear BDistal femoral physeal injury CPatellar dislocation DPatellar tendon rupture
C Patellar dislocation The patient's presentation is consistent with patellar dislocation. The patella tracks along the trochlear groove of the femur with knee flexion and extension. The patella can be displaced from the trochlea, most commonly in the lateral direction, by a twisting or valgus force on a flexed knee. A direct blow can also cause dislocation. Superior dislocation is rare but can occur with hyperextension of the knee or a direct blow. Medial dislocation is also rare and usually occurs after surgery to release the lateral patellar retinaculum. Risk factors for patellar dislocation include female sex, age younger than 20 years, and sports including gymnastics, dance, soccer, hockey, and weight lifting. Patients report a feeling of the knee buckling or giving way and may feel or hear a "pop." The knee will be fixed in flexion with the patella located laterally over the femoral condyle. X-rays show lateral displacement of the patella relative to the trochlea. The patella may spontaneously reduce with knee extension by the time the patient is seen, so the history of injury is important. If the patient presents with a dislocated patella, the diagnosis is usually apparent, and an attempt at closed reduction is indicated. The reduction maneuver involves flexing the hip to relax the quadriceps, extending the knee, and applying pressure on the lateral patella. If the dislocation has been present for several hours, sedation may be required. Physical examination findings after reduction include hemarthrosis presenting as a joint effusion, tenderness over the medial retinaculum and medial patellofemoral ligament, and a positive apprehension sign (the patient expresses pain and apprehension when the examiner pushes the patella laterally). X-rays should be performed or repeated after reduction and typically shows a joint effusion without bony abnormalities but may show osteochondral fractures or loose bodies. Initial treatment involves rest, ice, and bracing, followed by rehabilitation. Surgery to reconstruct the medial patellofemoral ligament or realign the patella may be indicated in certain patients.
A 72-year-old woman who underwent surgery for a left total knee arthroplasty 2 days ago presents to the emergency department with difficulty walking. She feels like she is constantly "stubbing" her toes on the left and complains of numbness and tingling in her left foot. On physical examination, she has decreased sensation to light touch over the dorsum of the foot at the first web space. She has one out of five strength (no movement but palpable muscle contraction) with ankle dorsiflexion and great toe extension. She has five out of five strength with these motions on the right. Deep tendon reflexes are normal. She has moderate swelling and ecchymoses over her knee, and her incision is intact and appears free of infection. Which of the following is the most likely diagnosis? A Compressive neuropathy of the sciatic nerve B Herniated lumbar disc at L4-5 C Peroneal nerve palsy D Saphenous nerve injury
C Peroneal nerve palsy The patient in the vignette above has a peroneal nerve palsy. The common peroneal nerve branches off the sciatic nerve in the popliteal fossa and winds anteriorly around the fibular neck, where it is superficial, so it is vulnerable to compression or traction injury. The superficial branch of the nerve supplies motor function to the muscles of the lateral compartment of the leg (peroneus longus and brevis) and sensation to anterolateral distal lower leg and the dorsum of the foot. The deep peroneal nerve supplies motor function to the tibialis anterior, extensor digitorum longus, peroneus tertius, extensor hallucis longus, extensor digitorum brevis, and extensor hallucis brevis and sensation to the first web space of the foot. The nerve can be compressed by pressure from lying on one's side for a prolonged period of time or by a tight dressing or cast. Crossing the legs or prolonged squatting can also cause nerve compression. Correction of a valgus deformity during knee arthroplasty surgery can cause a traction injury to the nerve. Patients typically present with a "foot drop" or inability to dorsiflex the foot and extend the toes. Muscle testing reveals weakness in ankle dorsiflexion and hindfoot eversion and decreased sensation over the dorsum of the foot. Patients may exhibit a "steppage gait," which involves lifting the foot up while walking to clear the toes. Patients may have paresthesias but typically do not have pain. Prevention of peroneal nerve palsy includes ensuring that postoperative dressings or splints are not too tight and positioning the patient to avoid pressure on the lateral side of the knee. If a peroneal palsy is identified in a postoperative patient, dressings around the knee should be removed immediately and the knee placed in slight flexion. Treatment is observation, as many patients will recover over three to six months. Most patients benefit from the use of an ankle-foot orthosis for walking and exercises for range of motion of the foot and ankle. Electromyography and nerve conduction tests can be helpful to evaluate the severity of nerve injury and provide a prognosis for recovery. Surgery to decompress the nerve is rarely indicated except when entrapment of the nerve is suspected.
A 17-year-old boy presents to the emergency department after suffering a neck injury while performing a tackle during a football game. He has been placed in a cervical collar. He is complaining of pain in his neck and is unable to move his extremities. On physical examination, he is found to be bradycardic and hypotensive. Imaging demonstrates a C5 cervical cord injury due to a C5-C6 fracture-dislocation. In addition to flaccid paralysis, what would you expect to find on initial examination? AHyperreflexia BPositive Babinski sign CPriapism DUrinary incontinence
C Priapism Priapism may be demonstrated in males with a complete spinal cord injury. The most common causes of traumatic spinal cord injury are motor vehicle collisions, falls, violence (gunshot wounds), and sports injuries. Injuries to the spinal cord are more common in men. Alcohol use is a common risk factor, as well as underlying spinal disease (cervical spondylosis, osteoporosis). A spinal cord injury occurs as a result of compression or contusion from an injury to the vertebra, often as a result of fracture or dislocation of a bony element, ligament tear, or intervertebral disc disruption. Edema of the spinal cord can result within hours after the injury. Patients will present with pain at the site of the fracture and quadriparesis or quadriplegia, depending on the site of the injury. In the acute stage, a patient with a complete cord injury will present with absent reflexes, flaccid muscle tone, priapism, and no response to plantar stimulation. As time goes on, patients develop spasticity and hyperreflexia as well as urinary retention and bladder distention. Incomplete spinal cord injuries have more varying degrees of motor and sensory function. Immediately after the injury, patients can develop spinal shock, causing bradycardia and hypotension. In the emergency setting, patients must be placed in a rigid cervical collar, and spinal movements must be minimized. Typically, plain radiographs of the cervical spine are initially ordered. A CT scan of the spine is obtained to assess for fracture. CT is preferred over magnetic resonance imaging for assessing fractures, however, magnetic resonance imaging should also be obtained to assess damage to the spinal cord. Patients with cervical cord injury, especially at higher levels, will often need intubation due to involvement of the phrenic nerve and intercostal muscles. Urinary catheterization is necessary to avoid bladder distension. These patients should be admitted to the intensive care unit for continuous monitoring of vital signs and regular neurological checks. Methylprednisolone has previously been suggested to improve neurologic outcomes in patients with traumatic spinal cord injury, however, its efficacy continues to be debated. The spinal cord injury should be decompressed and stabilized with neurosurgical intervention. Patients will need physical and occupational therapy.
A 55-year-old man with a history of myocardial infarction presents to the emergency department with dizziness and multiple episodes of syncope. He is bradycardic with a heart rate of 52, his blood pressure is 92/50, and he is diaphoretic. An electrocardiogram is ordered, which reveals constant PR intervals across the rhythm strip of the same length with occasional dropped wide QRS complexes. Which of the following is the most likely diagnosis? AFirst-degree atrioventricular block BSecond-degree Mobitz type I (Wenckebach) atrioventricular block CSecond-degree Mobitz type II atrioventricular block DThird-degree atrioventricular block
C Second-degree Mobitz type II atrioventricular block Mobitz type II heart block is a type of second-degree atrioventricular (AV) block, which is a disease affecting the conduction system of the heart where the atrial impulse through the AV node is delayed or blocked. Mobitz type II is specifically related to a problem in the infranodal conduction system (i.e., the His-Purkinje conduction system), and is usually seen in patients who have structural heart disease, history of myocardial ischemia, and myocardial fibrosis or sclerosis. In normal conduction, an electrical impulse from the sinoatrial node travels through the atria (seen as the PR interval on ECG), to the AV node, down the His-Purkinje system, to reach the ventricles creating a ventricular contraction (reflected as the QRS complex on ECG). Mobitz type I (also known as Wenckebach block), is the more common form of second-degree AV block and can be seen in patients without structural heart disease. It has a better prognosis than type II, since the latter has risk of progressing to a complete heart block, which has an associated risk of mortality. A main cause for AV block are cardioactive drugs (e.g., digoxin, beta-blockers, calcium channel blockers, certain antidysrhythmic drugs), and other drugs such as lithium, benzathine penicillin, and alpha agonists have been implicated in second degree AV block. This condition may also be associated with underlying diseases (e.g., inflammatory diseases, infiltrative disease, malignancies, metabolic and endocrine disorders, collagen vascular diseases), cardiac tumors, trauma, ethanol poisoning, and iatrogenic heart procedures. Some patients have a genetic predisposition, and may have underlying structural heart disease. An ECG is used to diagnose the presence and type of second-degree AV block. Mobitz II AV block is characterized by intermittent non-conducted P waves on a rhythm strip, and is not associated with a prolonged PR interval before the nonconducted atrial beat. Each P wave is associated with a QRS complex until a P wave appears that is not followed by a QRS. The QRS complex is often widened, and there may be an associated bundle branch or fascicular block. Patients with second-degree AV block may be asymptomatic (more commonly in patients with type I disease) or have a variety of symptoms such as lightheadedness, dizziness, and syncope. Some patients may experience palpitations, hypotension, diaphoresis, and bradycardia. If there is an associated myocardial ischemia or infarction, patients may present with chest pain. Those with Mobitz type II second-degree AV block should be admitted and closely monitored. Transcutaneous pacing should be started in all patients identified with this condition, including those who are asymptomatic, due to risk of progression to complete heart block, until permanent pacing can be initiated. Mobitz type II rhythm often does not respond to atropine, and is usually only reserved for type I
A 24-year-old woman presents to the emergency department with vaginal bleeding and pelvic pain. Her last menstrual period was six weeks ago. The urine pregnancy test is positive, and the transvaginal ultrasound shows no intrauterine pregnancy. Which of the following additional findings supports the diagnosis of ectopic pregnancy? A Heterogeneous and thickened endometrial cavity B Irregularly shaped intrauterine gestational sac C Serum human chorionic gonadotropin level of 3,600 mIU/m D Serum human chorionic gonadotropin level of 900 mIU/mL
C Serum human chorionic gonadotropin level of 3,600 mIU/m Ectopic pregnancies are extrauterine pregnancies that occur when the gestational sac implants outside of the uterus. Most ectopic pregnancies occur in the fallopian tubes, however, other possible sites of ectopic pregnancies include the cervix, ovaries, and cesarean scars. Risk factors for ectopic pregnancy include prior ectopic pregnancy (most important), prior pelvic inflammatory disease, use of intrauterine devices, and prior tubal surgeries. Ectopic pregnancies present most often with vaginal bleeding and pelvic pain during the first trimester of pregnancy. They most often present at six to eight weeks of estimated gestational age. The vaginal bleeding tends to be heavy and painful, but it can be light and intermittent or painless. Ruptured ectopic pregnancies can cause significant hemorrhage, resulting in symptoms related to hypovolemia, such as lightheadedness, syncope, palpitations, tachycardia, and hypotension. The diagnosis of ectopic pregnancy should be considered in women who present with vaginal bleeding and pelvic pain in the first trimester of pregnancy. The diagnostic tests used to confirm the diagnosis of ectopic pregnancy are the serum level of human chorionic gonadotropin and transvaginal ultrasound. An intrauterine gestational sac is normally visible on transvaginal ultrasound in pregnancies with a serum level of human chorionic gonadotropin of at least 3,500 mIU/mL. The serum level of human chorionic gonadotropin at which an intrauterine gestational sac should be seen on transvaginal ultrasound is known as the discriminatory zone. Some sources use 2,000 mIU/mL, while others use 3,500 mIU/mL (more sensitive) as the discriminatory zone. However, pregnant women often present with vaginal bleeding or pelvic pain in the first trimester and a serum human chorionic gonadotropin level less than 2,000 mIU/mL. In these patients, the serum human chorionic gonadotropin level can be reassessed after 48 to 72 hours. If the serum human chorionic gonadotropin level is decreased after 48-72 hours, then a spontaneous miscarriage is likely. If the serum human chorionic gonadotropin level is rising appropriately (by at least 35% every 48 hours or doubling every 72 hours), then the level should be rechecked every 48-72 hours until it is above the discriminatory zone. A transvaginal ultrasound can be repeated once the value is above the discriminatory zone. In patients who have a serum human chorionic gonadotropin level that is rising but not at the appropriate levels after three measurements, the pregnancy is considered abnormal. The differentials include ectopic pregnancy and a miscarriage. The treatment options for ectopic pregnancies include expectant management, methotrexate, and surgical intervention. Expectant management is not recommended for women with suspected ectopic pregnancy and serum human chorionic gonadotropin levels above 200 mIU/mL. Methotrexate therapy is recommended in patients with ectopic pregnancy who are reliable to follow up, are hemodynamically stable, have normal renal, hepatic, and hematologic function, have no fetal cardiac activity on ultrasound, and have pretreatment serum human chorionic gonadotropin levels below 5,000 mIU/mL. Surgical intervention is indicated in patients who meet one or more of the following criteria: hemodynamically unstable, suspected tubal rupture, heterotopic pregnancy with coexisting viable intrauterine pregnancy, contraindications to methotrexate, and failure to respond to methotrexate.
A 25-year-old man in South Florida presents during the summer with hypopigmented lesions on his upper back and proximal arms. The lesions do not itch or hurt, but he is bothered by the appearance of the lesions. What is the most likely diagnosis? APityriasis rosea BSeborrheic dermatitis CTinea versicolor DVitiligo
C Tinea versicolor Tinea versicolor is a common fungal skin infection that is caused by the Malassezia yeast. Risk factors include living in a tropical climate, immunosuppression, hyperhidrosis, and the use of topical oils. Tinea versicolor is not contagious. The clinical presentation of tinea versicolor varies. The lesions may be hypopigmented, hyperpigmented, or erythematous. The lesions are typically macules or thin plaques but can coalesce into larger patches. Most of the time, there are no associated symptoms, but the lesions may mildly itch. Tinea versicolor may primarily be a cosmetic concern for patients. The lesions are located on the trunk and upper extremities most often and less often on the face and neck. The diagnosis is suspected clinically but should be confirmed with a potassium hydroxide preparation. The classic potassium hydroxide preparation finding is hyphae and yeast cells, which may be described as having a spaghetti and meatballs appearance under a microscope. In one-third of cases, there is fluorescence seen with a Wood lamp. Topical antifungals are the first-line treatment. Possible agents include topical azoles (ketoconazole, clotrimazole, miconazole), topical ciclopirox, and topical selenium sulfide. Oral antifungals, such as itraconazole and fluconazole, can be used for cases with widespread involvement or in patients who fail topical therapy. It is important to educate patients that the abnormal skin pigmentation may not resolve until up to several months after successful treatment.
A patient with social anxiety disorder presents to the emergency department complaining of feeling diaphoretic and anxious following a large family dinner. Which of the following best describes why patients with social anxiety disorder feel fearful in social settings? A Fear of a specific object B Fear of a specific situation C Fear of being unable to find help in the event of an unexpected problem D Fear of embarrassment or ridicule from peers
D Fear of embarrassment or ridicule from peers Explanation: Patients with social anxiety disorder feel anxious specifically because they fear embarrassment or ridicule from peers. This disorder has an adult onset, and there is typically an inciting event where the patient was embarrassed or humiliated in front of others. Common inciting events are public speaking, using the bathroom in public, or eating in public. Phobias, in general, are one of the most common mental disorders in the United States and are characterized by an irrational fear and inappropriate levels of anxiety when a minimally provoking event occurs. Response to a potential stimuli or actual exposure to stimuli is excessive and may cause a full panic attack. As a result of the discomfort associated with this response, situations that are known to provoke a negative response are therefore avoided or endured with a considerable amount of discomfort or apprehension. Treatment of social anxiety disorder is selective serotonin reuptake inhibitors. Second-line treatment is short-term benzodiazepines.
A 57-year-old man with chronic obstructive pulmonary disease presents to the emergency room complaining of dyspnea, headache, and tremor. During initial observation, he appears anxious and confused with mild cyanosis of the oral mucosa. Vitals reveal HR 108 bpm, RR 23/min, BP 148/93 mm Hg, T 98.3°F, and SpO2 86% on room air. He is immediately started on high-flow supplemental oxygen via venturi mask. After five minutes of intervention, the patient demonstrates a lethargic state of consciousness and voices manifestations of delirium and paranoia. Repeat vitals indicate HR 110 bpm, RR 21/min, BP 152/95 mm Hg, T 98.7°F, and SpO2 97% on supplemental oxygen. Which of the following is the best next step? A Continue current rate of high-flow supplemental oxygen B Discontinue supplemental oxygen C Intubate and initiate mechanical ventilation D Reduce the flow of supplemental oxygen
D Reduce the flow of supplemental oxygen Acute respiratory failure is defined as a life-threatening dysfunction of the respiratory system that results in abnormalities of oxygenation or ventilation. Hypercapnia is an elevated arterial carbon dioxide and is a common finding in patients with hypoventilation or increased physiologic dead space and limited pulmonary reserve, which is seen in patients with chronic obstructive pulmonary disease. Respiratory failure presents with signs and symptoms of hypoxemia and hypercapnia. Dyspnea is the primary complaint resulting from hypoxemia with hypoxemic signs including cyanosis, restlessness, confusion, anxiety, delirium, tachypnea, bradycardia or tachycardia, hypertension, cardiac dysrhythmias, and tremor. Clinical manifestations of hypercapnia include dyspnea and headache with peripheral and conjunctival hyperemia, hypertension, tachypnea, tachycardia, impaired consciousness, papilledema, and asterixis. Initial assessment of a patient with suspected hypercapnia and impending respiratory failure should include assessment of the airway, breathing, and circulation. Peripheral saturation should be monitored, with administration of supplemental oxygen for levels less than 90%. In patients with chronic obstructive pulmonary disease, the ventilatory drive is altered due to a chronic state of hypoxemia secondary to dead space in the lungs. These patients then rely on the amount of circulating carbon dioxide to regulate respiration. The administration of oxygen, therefore, may increase the degree of hypercapnia and subsequent metabolic acidosis but should not be withheld in patients with respiratory failure and significant oxygen desaturation. Oxygen can be administered noninvasively via nasal cannula or venturi mask and should be titrated to achieve an SpO2 of 90-93%. Values higher than 95% require lowering of the supplemental oxygen flow. Noninvasive ventilation is most commonly implemented with bilevel positive airway pressure (BiPAP) or a bag-valve mask, with goals of reducing the partial pressure of arterial carbon dioxide and improved oxygenation. Intubation and mechanical ventilation are indicated in patients who develop severe respiratory acidosis (pH < 7.2) with or without a marked depression in the level of consciousness. Patients who require invasive ventilation techniques include those with severely impaired consciousness, an inability to protect their airway from excess secretions, hemodynamic instability, severe cardiopulmonary distress, or those with facial or esophageal injury or gastric bypass surgery. Laboratory studies include arterial blood gas, serum chemistry panels, bicarbonate, complete blood count, and electrolytes. If indicated, a toxicology screen, thyroid function test, or creatine phosphokinase test may reveal an underlying disorder. Chest radiograph can aid in the diagnosis of the causative disorder, with findings of chronic obstructive pulmonary disease including hyperinflation with flattened diaphragms. Sedatives should be reversed with antidotes (e.g., naloxone, flumazenil) if suspected as the cause of respiratory failure. Empiric therapies that target the underlying disorder should be implemented in a stable patient. For chronic obstructive pulmonary disease, these commonly include inhaled bronchodilators (albuterol, tiotropium) and corticosteroids (methylprednisolone).
A 33-year-old woman presents with recent-onset fever, abdominal cramps, and diarrhea. She describes frequent, small-volume, bloody diarrhea and reports no recent antibiotic use or history of gastrointestinal disorders. Vitals reveal HR 105 bpm, RR 19/min, BP 138/78 mm Hg, T 103.9°F, and SpO2 98% on room air. Which one of the following suspected organisms requires empiric antibiotic therapy? AClostridium perfringens BSalmonella CShiga-toxin-producing Escherichia coli DShigella
D Shigella Acute infectious diarrhea is defined as a diarrheal illness that lasts < 2 weeks and is caused by a virus, bacteria, or protozoan. Common causes of infectious diarrhea include Salmonella, Campylobacter, Shigella, Cryptosporidium, Shiga-toxin-producing Escherichia coli, Yersinia, Entamoeba histolytica, and Vibrio. Diarrhea can be severe or bloody, implicating bacterial pathogens, or watery, often caused by viral agents or protozoa. Shigella is a particularly virulent bacterium that causes dysenteric diarrhea and is associated with significant morbidity and mortality worldwide. It is spread via the fecal-oral route in industrialized countries and direct person-to-person contact or from contaminated food or water in resource-limited countries. Shiga toxin, produced by some strains of Shigella and E. coli, may result in hemolytic uremic syndrome in children. Common presentations of patients with Shigella gastroenteritis include high fever, abdominal cramps, tenesmus, and bloody mucoid diarrhea. Complications associated with this condition include proctitis, rectal prolapse, toxic megacolon, intestinal obstruction, and colonic perforation. Early diagnostic testing measures include microscopic evaluation of the stool, revealing white and red blood cells. Stool culture is the diagnostic test of choice, and all isolates should undergo susceptibility testing to ensure proper antibiotic selection. First-line treatment of Shigella-related diarrhea is supportive, with intravenous hydration indicated only if the patient cannot tolerate oral rehydration strategies. Antimotility agents (e.g., diphenoxylate, paregoric, loperamide) should be avoided as they may prolong fever, diarrhea, and bacterial shedding in the stool. Empiric antibiotic therapy is indicated only in symptomatic patients while awaiting confirmation of the diagnosis. In this subset of patients, antimicrobial agents have been shown to reduce the duration of fever and diarrhea by about 2 days and shorten the duration of bacterial shedding, which reduces the risk of person-to-person transmission. While antibiotic selection should be dictated by susceptibility reports, most regimens include a fluoroquinolone (levofloxacin, ciprofloxacin), azithromycin, a third-generation cephalosporin (cefixime, ceftriaxone), trimethoprim-sulfamethoxazole, or ampicillin. Oral rehydration using glucose-based formulations is important to replete fluids and electrolytes. Strategies for preventing transmission include frequent handwashing and avoidance of intercourse until 2 weeks after the diarrhea resolves. Conversion to negative stool cultures is usually evident after 48 hours of antimicrobial therapy, at which time patients who handle food can return to food preparation.
A 21-year-old baseball pitcher reports to the orthopaedic clinic complaining of anterior shoulder pain in his throwing arm that has progressed over the past three weeks. He states his pain is worse in the cocked throwing position, and he reports clicking, popping, and a catching sensation with throwing. He has a history of glenohumeral subluxation in this shoulder. Which of the following is the most likely diagnosis? ABiceps tendinitis BMultidirectional shoulder instability CRotator cuff tear DSuperior labrum anterior posterior tear
D Superior labrum anterior posterior tear Glenoid labral tears occur secondary to acute injuries to the glenohumeral joint. The glenoid labrum is a fibrocartilaginous disc that serves to deepen the articulation between the humeral head and the glenoid fossa of the scapula. A superior labrum anterior posterior tear is a glenoid labral tear that involves the superior portion of the labrum and progresses from anterior to posterior in a curved fashion. The most common mechanism of injury is an inferior traction-type injury caused from a fall or a sudden pull when lifting a heavy object. Throwing and overhead athletes are more likely to sustain a superior labral anterior posterior tear due to the increased stress on the glenoid labrum with repetitive abduction and external rotation during the cocking phase. Other mechanisms that may cause a labral tear include traumatic glenohumeral dislocation or a direct blow to the shoulder. Patients with labral tears caused by repetitive overuse complain of anterior shoulder pain with episodic clicking and mechanical symptoms, such as catching and locking. These symptoms are aggravated in the cocked throwing position. Athletes may present with a decline in function or throwing velocity. Physical examination should investigate the integrity and tenderness of the proximal biceps tendon and assess the degree of restricted internal rotation and excessive external rotation of the glenohumeral joint. Patients with a superior labrum anterior posterior tear commonly demonstrate unilateral scapulothoracic dysfunction. Special tests used to guide diagnosis of glenoid labral tears include the anterior glide test, compression rotation test, O'Brien test, crank test, and Speed test. Diagnostic testing is best accomplished using 3-Tesla magnetic resonance arthrogram scanning. Plain radiographs cannot diagnose soft tissue injury but are useful to rule out concomitant injuries. Shoulder arthroscopy is the gold standard to diagnose superior labrum anterior posterior tears with direct visualization of the injury. Orthopaedic referral should be obtained prior to performing advanced imaging studies in most cases and is warranted in the evaluation and treatment of all overhead throwing athletes or patients with overhead repetitive work. Conservative treatment is initially implemented with rest and the use of nonsteroidal anti-inflammatory drugs or acetaminophen for pain control. Rehabilitation programs aim to improve glenohumeral and scapulothoracic motion and increase the strength and endurance of the rotator cuff and scapulothoracic muscles. Most patients are able to return to pre-injury participation after adequate rehabilitation. Surgical management may be needed if conservative management fails to reduce pain and improve shoulder function. Postsurgical recovery requires six to 12 months before athletes can return to throwing activity.
A 42-year-old man presents to the emergency department complaining of unbearable, diffuse abdominal pain that radiates to his right shoulder and began one hour ago. His medication list includes hydrochlorothiazide 25 mg daily for essential hypertension, aspirin 81 mg daily for coronary artery disease, and ibuprofen 800 mg daily for the past three weeks for an ankle sprain. Vital signs demonstrate HR 105 bpm, RR 19/min, T 95.2°F, BP 131/87 mm Hg, and SpO2 99% on room air. Physical exam reveals a weak radial pulse with cool distal extremities and abdominal rigidity. Which of the following is the most appropriate initial diagnostic study? AAbdominal ultrasound BComputed tomography of the abdomen CMagnetic resonance imaging of the abdomen DUpright chest radiography
D Upright chest radiography Peptic ulcer disease results from ulceration of the muscularis mucosa through the deeper layers of the bowel. These lesions occur in the gastrum or the duodenum and are most commonly caused by nonsteroidal anti-inflammatory drug use and Helicobacter pylori infection. Patients with comorbid disease, advanced age, and more severe physiologic insult (e.g., hypotension, metabolic acidosis, acute kidney injury, hypoalbuminemia) are likely to have poor outcomes from complicated peptic ulcer disease. Perforated gastric ulcers have a worse prognosis than duodenal ulcer perforation. Three phases characterize the clinical manifestations of ulcer perforation. Within two hours of onset, the initial phase is defined by sudden, severe, diffuse abdominal pain. Patients may complain of radicular pain to the right shoulder or bilateral shoulders. Signs in this phase include tachycardia, a weak pulse, cool extremities, a low temperature, and progressive abdominal rigidity. After 2 to 12 hours, abdominal pain may regress. Physical examination demonstrates marked abdominal rigidity. Findings may also include absence of liver dullness to percussion due to peritoneal air, pelvic peritoneum tenderness, and right lower quadrant tenderness. The third phase, which presents more than 12 hours after initial onset, is primarily characterized by abdominal distention. Findings on physical examination and laboratory testing can indicate hypovolemia and hyperthermia. Plain film radiography is the first diagnostic imaging used to identify a perforated peptic ulcer. An upright chest radiography should be performed to detect free air in the abdomen, which is highly indicative of perforated viscus. If this finding is not evident, CT or ultrasound can be used to detect small amounts of free air or fluid. Patients with perforated peptic ulcers require supportive care, such as taking in nothing by mouth and fluid resuscitation. Acid-suppressive therapy should be initiated in the form of an intravenous proton pump inhibitor. Omeprazole is the first line in this class, but pantoprazole and esomeprazole are acceptable alternatives. Patients should be switched to an oral formulation once they are able to tolerate oral intake. Early diagnosis and treatment of H. pylori dramatically reduces recurrent ulcers and complications. The treatment regimen for patients without risk factors for macrolide resistance (prior exposure to macrolide therapy, high local clarithromycin resistance rates) includes a proton pump inhibitor, amoxicillin, and clarithromycin for 14 days. Those at risk for macrolide resistance should be administered a proton pump inhibitor, bismuth subsalicylate, metronidazole, and tetracycline for 14 days. Offending nonsteroidal anti-inflammatory drugs (including aspirin) should be discontinued. If this is not a feasible option, patients may be switched to a COX-2 inhibitor, such as celecoxib. Patients with uncontrolled hemorrhage or continued leakage from a perforated ulcer require urgent or emergent surgery. Prognosis is excellent if the perforated ulcer is identified and properly treated within the first six hours.
A 68-year-old man presents to the emergency department with a severely painful and rigid erection. He is being treated for erectile dysfunction with sildenafil. He denies any prior history. Physical examination reveals a penile erection that has been present for five hours. Cavernosal blood gas analysis shows hypoxemia, hypercarbia, and acidemia. Which of the following is the most appropriate next step in management? A Urology consult and intracavernosal injection with phenylephrine alone B Urology consult and observation C Urology consult and shunt surgery D Urology consult, intracavernosal injection with phenylephrine, and aspiration
D Urology consult, intracavernosal injection with phenylephrine, and aspiration Priapism is the persistent erection of the penis or clitoris that is not associated with sexual desires and typically lasts at least four hours. Priapism may occur in any age group and is common in individuals with sickle cell disease. Priapism has a bimodal peak distribution of incidence, first occurring in children between 5 to 10 years and then in adults between 20 to 50 years. Priapism predominantly affects the corpora cavernosa but may also affect the corpus spongiosum and the glans penis. Nitric oxide plays a fundamental role in the erectile response by relaxing penile smooth muscle. There are two types of priapism: ischemic (urologic emergency, more common) and nonischemic (self-limiting). Ischemic priapism is associated with failure of detumescence due to impaired relaxation and paralysis of cavernosal smooth muscles. Nonischemic priapism results from penile or perineal trauma, typically due to needle injury to the cavernosal artery. Causes of priapism may be classified as idiopathic (primary priapism) or secondary priapism, which includes medications (such as anticoagulants, antihypertensives, antidepressants, phosphodiesterase type 5 inhibitors, alpha-blockers, intracavernous injections, atomoxetine, and cocaine), spinal shock, sickle cell disease, malaria, spider toxins, gout, diabetes, and amyloidosis. Patients with ischemic priapism present with a painful and rigid erection. Rarely, they may present with penile gangrene and necrosis of the entire penis. Penile pain and rigidity are less intense in individuals presenting with nonischemic priapism most commonly after a urologic procedure or penile trauma. History and physical examination may aid in distinguishing between the types of priapism and should include information about the duration of erection, prior history, medications, recreational drug use, penile or perineal trauma, sickle cell disease, and severity of penile pain. When the type of priapism is uncertain, cavernosal blood gas analysis should be performed, which may show hypercarbia, hypoxemia, and acidemia in ischemic priapism and normal levels of oxygen, carbon dioxide, and pH in nonischemic priapism. Alternatively, color Doppler ultrasound of the penis may be performed to differentiate ischemic from nonischemic priapism. Complete blood count may be performed if there are concerns about blood dyscrasias. Urine toxicology should be performed in the setting of suspected or known drug use or psychiatric history. Management of priapism depends on the type and duration of priapism and typically involves urological consultation. For ischemic priapism lasting fewer than four hours, intracavernosal injection of a sympathomimetic drug (e.g., phenylephrine) should be performed. If an erection lasts more than four hours, aspiration with or without irrigation should be added to intracavernosal injection. Repeat aspiration and injections over several hours may be necessary to achieve detumescence. Ischemic priapism that is refractory to repeat aspirations and injections should be treated by shunt surgery. Once ischemic priapism has been ruled out, observation alone may be appropriate as initial management of nonischemic priapism as it can spontaneously resolve without treatment. In patients who prefer intervention rather than observation, arteriography with embolization may be indicated.
What is the primary driver of proliferation and differentiation of red blood cell progenitors?
Erythropoietin
What is the etiology of pityriasis rosea?
HHV-7 aka viral
What medications can cause idiopathic thrombocytopenic purpura?
Gold, quinidine, ranitidine, furosemide, and nonsteroidal anti-inflammatory drugs.
What other injury can occur concomitantly with a glenoid labral tear after shoulder dislocation?
Hill-Sachs lesion, which is a fracture of the posterolateral humeral head.
A 45-year-old Native American woman presents to the emergency department with fever, jaundice, and right upper quadrant pain for six hours. Her pain is constant and she denies any nausea or vomiting. An abdominal ultrasound reveals the presence of a common bile duct stone with biliary duct dilatation but no other abnormal findings. Which of the following is the most likely diagnosis? AAcute cholangitis BAcute cholecystitis CAcute viral hepatitis DBiliary colic
A Acute cholangitis Acute cholangitis is a bacterial infection of the biliary tree due to an obstruction in the area, most commonly by a gallstone. Obstruction can also be due to a neoplasm or stricture. When the biliary tract is obstructed, intraluminal pressure is elevated, leading to an optimal environment for the bile (which is normally sterile) to become infected. Cholangitis will not typically develop without an obstruction. The infection can ascend from the tree into the hepatic ducts, hepatic veins, and perihepatic lymphatics leading to bacteremia. The most common organisms isolated in these cases ranging from most common to least common are Escherichia coli, Klebsiella species, Enterococcus species, Streptococcus species, Enterobacter sepsis, and Pseudomonas aeruginosa. There may be polymicrobial infection found in bile and blood cultures. Primary sclerosing cholangitis is a type of cholangitis that is a chronic liver disease associated with inflammation and fibrosis of the biliary tree and hepatic ducts. Choledocholithiasis (gallstones in the common bile duct) is the most common cause of acute cholangitis. Other less common causes are obstructive tumors (e.g., pancreatic cancer, cholangiocarcinoma), strictures, stenosis, endoscopic manipulation, sclerosing cholangitis, and after endoscopic retrograde cholangiopancreatography. Other risk factors include recent cholecystectomy, history of cholangitis, endoscopic manipulation of the biliary tree, and history of human immunodeficiency virus or acquired immunodeficiency syndrome. Patients with cholangitis will have a spectrum of presentations, ranging from mild symptoms to fulminant sepsis. Classic findings associated with this condition are right upper quadrant pain, fever, and jaundice (Charcot triad). If there are mental status changes and sepsis involved, these are grouped together as a Reynolds pentad. Other symptoms include chills, abdominal pain, pruritus, acholic stools, and malaise. Physical exam findings include fever, right upper quadrant tenderness, hepatomegaly, jaundice, hypotension, tachycardia, and rarely, peritonitis. Ultrasonography and computed tomography scanning are first-line imaging modalities. Transabdominal ultrasound is very sensitive and specific for examining the gallbladder and assessing bile duct dilatation and is very advantageous due to ease of use and lack of radiation. Computed tomography shows a more complete picture of the biliary tree and the surrounding structures and is able to detect radiolucent stones, as well as other complications of cholangitis (e.g., tumors, liver abscesses). Endoscopic retrograde cholangiography, magnetic resonance cholangiopancreatography, and endoscopic ultrasonography are often used for diagnostic and therapeutic purposes. Endoscopic retrograde cholangiography is considered the criterion standard for imaging the biliary tree and is reserved for patients who require therapeutic intervention. Laboratory studies such as complete blood count (which will reveal leukocytosis), liver function tests (which will reveal hyperbilirubinemia and elevated alkaline phosphatase), and blood cultures should be performed. Treatment depends on the severity of the patient's symptoms. Unstable patients should be closely monitored and stabilized with oxygen and intravenous fluids. Parenteral broad-spectrum antibiotics should be started empirically after blood cultures of drawn. In stable patients, medical treatment with parenteral antibiotics may be all that is required, with elective surgery being an option in the future. For those who do not respond to medical therapy or those who are severely ill, immediate emergency decompression via endoscopic retrograde cholangiopancreatography, sphincterotomy, or percutaneous drainage of the biliary tree may be required. Complications of cholangitis include liver failure, hepatic abscesses, sepsis, and acute renal failure.
A 45-year-old man presents to the emergency department with a dull discomfort in the groin that he noticed while he was powerlifting at the gym. Physical examination reveals a bulge in the right inguinal region within the Hesselbach triangle that is aggravated by Valsalva. It does not descend into the scrotum. Which of the following anatomic structures is most likely affected? A External ring B Femoral ring C Internal inguinal ring D Umbilical ring
A External ring A hernia is a protrusion of an organ or part of an organ through the wall that normally contains it. Inguinal hernias may result congenitally or may be due to a weakness in the floor of the inguinal canal. Hernias are more common in men than women and more prevalent in Caucasian individuals than non-Caucasian individuals. Inguinal hernias have a bimodal age distribution occurring in patients < 1 year of age or those > 40 years of age. Risk factors for developing hernias include a history of prior hernia, older age, male sex, chronic cough, chronic constipation, abdominal wall injury, smoking, and family history of hernias. Hernias may be classified as congenital (due to failure of processus vaginalis to close) or acquired (due to weakening or disruption of the fibromuscular tissues of the body wall). Additionally, inguinal hernias may be classified as direct or indirect. Direct inguinal hernia protrudes medially to the inferior epigastric vessels within the Hesselbach triangle through the external ring. Indirect inguinal hernia, the most common type of inguinal hernia, protrudes at the internal inguinal ring, a site where the spermatic cord (in males) and the round ligament (in females) exit the abdomen. Patients with hernias usually present with a dull discomfort or heaviness in the groin, which may or may not be associated with a visible bulge. Discomfort is aggravated by straining, heavy lifting, or prolonged standing. Moderate to severe groin pain should raise the suspicion of an incarcerated or strangulated hernia. Physical examination typically reveals a bulge in the groin more predominantly on the right side but may also show no visible bulge. Bulge may be accentuated by Valsalva or cough and may be painful. In the setting of an incarcerated or strangulated hernia, pain is typically severe. Absence of a visible bulge in patients with high clinical suspicion should warrant additional maneuver including the invagination of some of the redundant scrotal skin into the inguinal canal. Incarcerated and strangulated hernias may result in ischemia and necrosis of hernia contents. Diagnosis of direct inguinal hernia is typically made clinically without the need for imaging. When the diagnosis is not apparent, ultrasound of the groin may be obtained. Computed tomography, magnetic resonance imaging, or herniorrhaphy may be useful when ultrasound does not establish a diagnosis. Initial treatment of uncomplicated hernia may be watchful waiting or manual reduction. In the setting of an incarcerated or strangulated hernia, urgent surgical repair performed within four to six hours of the onset of symptoms is indicated. Definitive treatment is by surgical repair.
Which of the following is most likely to confirm the diagnosis of active Mycobacterium tuberculosis? AChest radiograph BFluorochrome staining CNucleic acid amplification testing DTuberculin skin testing
C Nucleic acid amplification testing Tuberculosis is a pulmonary infection caused by Mycobacterium tuberculosis that manifests as primary tuberculosis, latent tuberculosis, reactivation tuberculosis, laryngeal tuberculosis, endobronchial tuberculosis, or tuberculoma. Tuberculosis is one of the world's most widespread and deadly diseases, affecting disproportionately disadvantaged populations (those experiencing homelessness and those who are malnourished), individuals infected with HIV, and those living in crowded and substandard housing. Tubercle bacilli transmitted by airborne droplets may be ingested by alveolar macrophages. Infection in the lungs ensues if the inoculum escapes the microbicidal activity of the alveolar macrophage, leading to primary tuberculosis. T-cells and macrophages mobilized by the body's immune system surround the organism in granuloma limiting their multiplication and spread. When this process occurs, the infection becomes contained (not eradicated), leading to latent tuberculosis infection. Reactivation tuberculosis occurs in patients with latent tuberculosis who did not receive preventive therapy or in individuals with latent tuberculosis who also have diabetes mellitus, silicosis, and an impaired immune system (e.g., those with HIV or those receiving corticosteroid therapy, tumor necrosis factor inhibitors, or other immunosuppressive drugs). Patients with latent tuberculosis are asymptomatic. With the reactivation of latent tuberculosis, these patients may present with slowly progressive malaise, anorexia, weight loss, night sweats, hemoptysis, chronic cough, and fever. Physical examination may reveal a chronically ill or malnourished patient, post-tussive apical rales, clubbing, or erythema nodosum but may also be normal. The approach to diagnosis of tuberculosis begins with history and physical examination to assess the patient's risk for tuberculosis. Patients meeting clinical criteria should undergo chest radiograph that may show ghon focus in patients with primary tuberculosis and upper lung lobe cavitary lesions in patients with active or reactivation tuberculosis. Lower lobe infiltrates with pleural effusion may be seen in the older population. A miliary pattern typically occurs when the organism has hematologically or lymphatically spread. In individuals with late-stage HIV, mediastinal lymph nodes may be present on a radiograph. If imaging suggests tuberculosis of the lungs or airways, three sputum specimens (obtained via cough or induction at least 8 hours apart and including at least one early-morning specimen) should be submitted for acid-fast bacilli smear, mycobacterial culture, and nucleic acid amplification testing. Hematology and biochemistry laboratory studies are typically normal in patients with pulmonary tuberculosis. The traditional approach to testing of latent tuberculosis is by tuberculin skin testing. Individuals vaccinated with bacillus Calmette-Guérin should be screened for latent tuberculosis using interferon-gamma release assay. The definitive diagnosis of tuberculosis is by the recovery of Mycobacterium tuberculosis by acid-fast bacilli smear or culture or nucleic acid amplification testing. Positive nucleic acid amplification test (with or without acid-fast bacilli smear or culture) is considered sufficient for diagnosis of tuberculosis. Treatment of latent tuberculosis involves the use of isoniazid for 6 months, rifampin for 4 months, or isoniazid plus rifapentine for 3 months. Active or reactivated tuberculosis can be treated using the RIPE regimen (rifampin, isoniazid, pyrazinamide, and ethambutol) for 4 months followed by rifampin and isoniazid for 2 months. In individuals with HIV who also have tuberculosis, a longer duration of therapy using the RIPE regimen should be considered. In these patients, drug interactions between rifamycin derivatives and some protease inhibitors should also be considered.
A 65-year-old man with hepatocellular carcinoma presents to the emergency department with acyanotic skin. He denies tobacco use. Physical examination reveals acrocyanosis but is otherwise normal. Laboratory studies show a white blood cell count of 7,500 cells/mm3, hematocrit 56%, hemoglobin 17.5%, and platelet count 250,000/microL. Serum erythropoietin is found to be elevated. JAK2 mutation test is negative. Which of the following is the most likely diagnosis? A Essential thrombocytosis B Polycythemia vera C Secondary polycythemia D Spurious polycythemia
C Secondary polycythemia Polycythemia refers to an abnormal elevation in hematocrit (> 49% in men, > 48% in women) or hemoglobin concentration (> 16.5 g/dL in men, > 16 g/dL in women) in peripheral blood. Absolute polycythemia is an increase in red blood cell mass and may be classified as primary or secondary. Primary polycythemia refers to an increase in red blood cell mass that stems from an acquired or inherited mutation (such as mutations in the VHL gene, EPO receptor, and JAK2 mutation) in the red blood cell progenitor cells. Causes of primary polycythemia include polycythemia vera (most common), primary familial and congenital polycythemia, Chuvash polycythemia, and congenital methemoglobinemia. Secondary polycythemia refers to an increase in red blood mass due to elevated serum erythropoietin caused by a physiologic response to tissue hypoxia (most common cause) or by a tumor. Causes of secondary polycythemia include chronic pulmonary disease, right-to-left cardiac shunt, sleep apnea, Pickwickian syndrome, and erythropoietin-secreting tumors, such as hepatocellular carcinoma, renal carcinoma, and pheochromocytoma. Patients with primary polycythemia are typically asymptomatic but may present with hyperviscosity symptoms (such as chest or abdominal pain, myalgia, weakness, fatigue, blurred vision, or a sense of depersonalization), thrombosis or bleeding, unexplained weight loss, fever, facial plethora, erythromelalgia, or aquagenic pruritus (pruritus after bathing). Patients with secondary polycythemia typically present with acyanotic skin, headaches, lethargy, or confusion. Physical examination may reveal plethora manifesting as redness of palms and acrocyanosis. Complete blood count is usually the initial diagnostic tool obtained and may reveal elevated hemoglobin and hematocrit. When repeated hematocrit exceeds 52% in men and 47% in women, red blood mass and plasma volume using nuclear scintigraphy should be performed. Serum erythropoietin in a patient with secondary polycythemia is typically normal or elevated. Measurement of arterial saturation typically reveals saturations less than 92%. Computed tomography of the abdomen may be obtained if an erythropoietin-producing tumor or kidney-associated cause is suspected. Asymptomatic patients with polycythemia may be observed. When symptoms suggestive of polycythemia vera or another myeloproliferative neoplasm develop, patients may be treated with hydration, phlebotomy, aspirin, or hydroxyurea. Secondary polycythemia may be treated by phlebotomy or by treating the underlying cause. Erythropoietin-producing tumors causing secondary polycythemia may be surgically resected.
A 60-year-old man presents to the emergency department with confusion, disorientation, delirium, and muscle weakness. His urine is very concentrated, and he is diagnosed with syndrome of inappropriate antidiuretic hormone secretion. Which of the following electrolyte abnormalities would explain his symptoms? AHyperkalemia BHypernatremia CHypokalemia DHyponatremia
D Hyponatremia Syndrome of inappropriate antidiuretic hormone secretion (SIADH) is caused by continued secretion of antidiuretic hormone or ADH (also known as arginine vasopressin or AVP), despite normal or increased plasma levels. ADH works locally on the renal collecting duct to cause reabsorption of water and works as a pressor systemically to cause arterial vasoconstriction to raise blood pressure. An inappropriate secretion of ADH leads to impaired water excretion, resulting in hyponatremia and hypo-osmolality (urine will be reduced in volume and very concentrated). Hypersecretion of ADH from the hypothalamus, ectopic production (e.g., neoplasia), drug stimulation (e.g., tricyclic antidepressants, barbiturates, opioids), or infection of the central nervous system or lungs can cause SIADH. Symptoms depend on the degree of hyponatremia and the rate at which the hyponatremia occurs, but the severity of symptoms is not directly correlated with the severity of hyponatremia. A rapid drop in serum sodium causes more symptoms than progressive decline in sodium levels, and patients may present with confusion, disorientation, delirium, muscle weakness, tremor, seizures, and even coma. Since there is no single test to diagnose this condition, diagnosis is made using the Bartter-Schwartz criteria that include the following findings: hyponatremia with hypo-osmolality, continual renal sodium excretion, urine that is less than maximally dilute, absence of clinical evidence of volume depletion (e.g., normal blood pressure, no skin turgor), no other cause of hyponatremia found (e.g., hypothyroidism, adrenal insufficiency, heart failure), and fluid restriction causes correction of hyponatremia. Hyponatremia (serum sodium < 135 mmol/L) in the presence of hypo-osmolality (serum osmolality < 280 mOsm/kg) is the hallmark of SIADH. Urinary sodium loss will be significant despite significant hyponatremia in patients with SIADH who are not on a sodium-restricted diet. Normally in the setting of hyponatremia, ADH should be shut down, causing maximally dilute urine. In patients with SIADH, urinary osmolality will be submaximally dilute, which is characteristic of SIADH. Serum bicarbonate and serum potassium will remain within reference range. Uric acid level may be decreased. It is important to rule out other causes of hyponatremia, such as renal disease, heart failure, liver failure, thyroid disorders, and adrenal insufficiency. Therefore, the following tests should also be ordered: serum cortisol, serum creatinine, blood urea nitrogen, blood glucose, urine osmolality, and thyroid-stimulating hormone. Treatment of SIADH depends on the degree of hyponatremia, sympomaticity, acuity of the syndrome (whether acute, beginning less than 48 hours ago, or chronic), urine osmolality, and creatinine clearance. Patients who have chronic SIADH will have unknown duration of hyponatremia and will be asymptomatic. The mainstay of treatment for chronic patients is fluid restriction. Urea is a solute excreted by the kidneys that can also be used as a treatment for chronic patients long-term to increase urine volume but is used with caution in patients with kidney disease. Additional treatments include loop diuretics (e.g., furosemide) with increased salt intake, mannitol, and demeclocycline. Correction of hyponatremia is warranted in patients who have severe symptoms (e.g., seizures, coma, respiratory arrest) despite the degree of hyponatremia and those who have acute moderate-to-severe hyponatremia within the last 48 hours. The goal in acute patients is to correct the hyponatremia without causing neurologic complications (e.g., central pontine myelinolysis) using 3% hypertonic saline, loop diuretics, vasopressin-2 receptor antagonists (e.g., conivaptan), and water restriction. The rate of correction should be less than 8 mEq/L in any 24-hour period and patients should be reassessed every two to four hours until stable. If SIADH is drug-induced, resolution occurs after discontinuing the offending agent.
What is the acronym for the predisposing factors to chronic pancreatitis?
TIGAR-O: toxic-metabolic, idiopathic, genetic, autoimmune, recurrent and severe acute pancreatitis, and obstructive.
What drugs or drug classes stimulate antidiuretic hormone release?
Antineoplastic agents, barbiturates, bromocriptine, haloperidol, histamine, opiates, nicotine, and tricyclic antidepressants.
When does a human chorionic gonadotropin level typically peak during pregnancy?
Approximately 10 weeks
A 22-year-old woman with normal body mass index presents to the emergency department complaining of weakness and fatigue. She is found to be hypokalemic and alkalotic, with dental enamel erosions. Upon questioning, the patient reports binge eating and self-induced vomiting at least twice daily for the past 6 months. Which of the following represents the best choice of pharmacotherapy for this patient after electrolyte imbalances have been corrected? AAmitriptyline BBupropion CFluoxetine DTopiramate
C Fluoxetine Bulimia nervosa is an eating disorder characterized by excessive calorie intake, followed by self-induced purging of calories (via vomiting, diuretics, laxatives, fasting, or excessive exercise). Patients with bulimia nervosa have an altered self-image that is overly concerned with weight and body shape. For diagnosis, the binging and purging episodes must occur at least once weekly for 3 months or longer. During an episode of binge eating, the patient may describe feeling out of control. The exact etiology of bulimia nervosa is not known, although disordered serotonin pathways play a role. Women are more likely to suffer bulimia nervosa than men, and the average age of onset is 20 years. Patients with bulimia nervosa often have comorbid psychological disorders, such as major depressive disorder, anxiety disorders, post-traumatic stress disorder, and body dysmorphic disorder. Cutting and other nonsuicidal self-injurious behaviors are often seen in patients with bulimia nervosa. Complications of bulimia nervosa include electrolyte imbalances, dehydration, dental erosions, Mallory-Weiss esophageal mucosal lacerations, ipecac-induced myopathy, endocrine dysfunction, and cardiac dysrhythmias (rare). Diagnosis of bulimia nervosa is clinical, as described above, based on the Diagnostic and Statistical Manual of Mental Disorders, volume 5, guidelines. The binge episode must include excessive calorie consumption in a discrete period of time, accompanied by feelings of loss of control. The eating disturbance must not occur exclusively during an episode of anorexia nervosa. Treatment of bulimia nervosa is with a three-pronged approach of nutritional rehabilitation, psychotherapy, and pharmacotherapy. Either of the three may be used alone, but a combination of all three has proven most effective. In nutritional rehabilitation, the patient is encouraged to consume an appropriate amount of calories throughout the day, including three meals and two snacks. Psychotherapy focuses on body image, feelings of loss of control, and concomitant depression, anxiety, or other psychological disturbances. Pharmacotherapy with antidepressants has proven effective in the treatment of bulimia nervosa, possibly due to the disruption of serotonin pathways found in the illness. Selective serotonin reuptake inhibitors, such as fluoxetine, are first-line therapy due to their minimal effects on weight and favorable side effect profile. Patients should be observed closely during the first few weeks of treatment, as this class of medicine is associated with an increased risk of suicidality in young patients.
A 45-year-old woman presents to the emergency department with recurrent, spontaneous bruising and nosebleeds. Physical examination reveals purpura and petechiae on the lower extremity bilaterally but is otherwise normal. Laboratory studies reveal a platelet count of 20,000/mcL and normal prothrombin and activated partial thromboplastin times. Which of the following is the most likely diagnosis? AHemolytic uremic syndrome BHeparin-induced thrombocytopenia CIdiopathic thrombocytopenic purpura DThrombotic thrombocytopenic purpura
C Idiopathic thrombocytopenic purpura Idiopathic thrombocytopenic purpura (also called primary immune thrombocytopenia or immune thrombocytopenic purpura) is an autoimmune bleeding disorder in which pathogenic antibodies bind platelet antigens, such as glycoproteins IIb/IIIa and Ib/IX, thereby facilitating their clearance from the circulation. The principal mechanism of idiopathic thrombocytopenic purpura involves specific immunoglobulin G produced by the B cell. Idiopathic thrombocytopenic purpura may be acute or chronic and primary or secondary. The acute form that is self-limiting is primarily seen in children after a viral illness or immunization. The prevalence of the chronic form is significantly higher in adults, especially women of childbearing age. While idiopathic thrombocytopenic purpura may be primary in most adults, it may be associated with connective tissue diseases (such as systemic lupus erythematosus), lymphoma, medications, and infections (such as the hepatitis C virus and HIV). Clinical manifestations depend on the platelet count and may include spontaneous bruising and nosebleeds, gingival bleeding, purpura, petechiae, menorrhagia, and intracranial bleeding in severe cases. Laboratory studies typically show isolated thrombocytopenia, normal prothrombin time, and normal activated partial thromboplastin time but may also show anemia in the setting of bleeding. Bone marrow may show elevated megakaryocytes but is rarely needed for diagnosis unless in patients with unexplained cytopenias in two or more lineages, patients older than 40 years, or in those who have failed the initial therapy for idiopathic thrombocytopenic purpura. Treatment is reserved for patients with a platelet count of 20,000 to 30,000/mcL or those with significant bleeding. The initial treatment of new-onset idiopathic thrombocytopenic purpura in adults is a short course of steroids with or without intravenous immunoglobulin or anti-D immunoglobulin. In children, watchful waiting or pharmacologic intervention using methylprednisolone or intravenous immunoglobulin may be used depending on the severity of the bleeding symptoms. Platelets may be transfused in the setting of active bleeding. Splenectomy is indicated in refractory cases. For adult patients with chronic idiopathic thrombocytopenic purpura who have failed corticosteroid therapy, immunoglobulin therapy, and splenectomy, romiplostim and eltrombopag are indicated.
What is the most common pulmonary symptom of tuberculosis?
Chronic cough.
Which cranial nerve is the most vulnerable to the effects of increased intracranial pressure?
Cranial nerve VI (abducens nerve) due to its long course in the subarachnoid space.
What medication increases the risk of vertebral fracture after cessation?
Denosumab
What is a 2:1 atrioventricular block?
It is a Mobitz-type rhythm with a fixed conduction ratio of 2:1 (two conducted beats to one non-conducted beat). It is difficult to identify on ECG as type I or type II.
What is crossed straight leg raise?
It is a procedure to test for pain radiating down the affected leg when the contralateral leg is raised. It is very specific for sciatica.
What is the likely Guillain-Barré variant in a patient presenting with ophthalmoplegia, ataxia, and areflexia?
Miller Fisher syndrome
What is Lutembacher syndrome?
The combination of mitral stenosis and an atrial septal defect.
In which nostril is intranasal sumatriptan given for cluster headache?
The nostril contralateral to the cluster headache.
What is the most common site of open fracture?
The tibia.
What is the inheritance pattern of von Willebrand disease?
autosomal dominant
What kind of bacteria are found in the genus Shigella?
Nonmotile, facultatively anaerobic, gram-negative rods.
A 31-year-old man reports to the emergency department after falling from a ladder. His left lower leg has two obvious deformities with a 2.1 cm open wound at one area of angulation and another 1.4 cm open wound at a distal site of concern. His past medical history includes hypertension and diabetes mellitus, he has no allergies. Which of the following medications is indicated for infection prophylaxis? ACefazolin BClindamycin CLevofloxacin DVancomycin
A Cefazolin Osteomyelitis is an infection of the bone that can occur secondary to contamination of an open fracture. Fractures that are more severe, with extensive soft tissue injury, damaged vascular supply, or a high degree of bacterial contamination, are more likely to lead to osteomyelitis. Patients who are immunocompromised or have chronic disease (e.g., diabetes mellitus, coronary heart disease, chronic renal failure) are more susceptible to infection. The most common organisms that cause osteomyelitis in patients with open fractures are Staphylococcus aureus, coagulase-negative staphylococci, and aerobic gram-negative bacilli. Pseudomonas, Aeromonas, and Vibrio species may be present if the wound came in contact with contaminated water. The most common clinical findings of osteomyelitis associated with fracture are poor wound healing and fracture nonunion. Classic signs of infection, such as fever, wound drainage, erythema, warmth, swelling, and pain, may also indicate an underlying bone infection. Open fractures are classified according to the Gustilo-Anderson grading system. Type I open fractures have a wound size < 1 cm with minimal contamination and comminution and adequate soft tissue coverage. Type II open fractures have a wound size > 1 cm with moderate contamination and comminution but no vascular injury that requires repair and adequate coverage of the fracture with soft tissue. Type III open fractures demonstrate severe contamination and severe comminution, segmental fractures, periosteal stripping, or a combination thereof. Type III open fractures can be further subcategorized as A, B, or C according to the degree of soft tissue coverage. Any open fracture that requires repair of the vascular supply is a type IIIC fracture. If osteomyelitis is suspected, laboratory studies should include erythrocyte sedimentation rate, C-reactive protein, white blood cell count, and blood cultures. Diagnostic imaging to evaluate an open fracture for osteomyelitis is best performed using MRI, CT, or nuclear imaging. Osteomyelitis secondary to a contaminated open fracture is treated using surgical debridement with irrigation, bone biopsy for culture, antimicrobial therapy, and fracture fixation, as indicated. Osteomyelitis can be prevented by implementing prompt debridement and surgical fixation and with the administration of prophylactic antibiotics. Patients with type I or II fractures should be prescribed antibiotics with gram-positive organism coverage. Cefazolin is the most commonly used agent in these patients. Patients at increased risk for methicillin-resistant S. aureus include those who have been recently hospitalized, reside in a long-term care facility, have undergone recent surgery, or receive hemodialysis. Additionally, patients with HIV infection, injection drug use, and recent antibiotic use are more susceptible to methicillin-resistant S. aureus infection. Any patient at risk for this pathogen should be given vancomycin or other agents with effective coverage. Agents used to treat type III fractures should additionally target gram-negative organisms with regimens commonly consisting of cefoxitin, cefotetan, or ampicillin-sulbactam. A patient with a type III fracture who has had potential exposure to contaminated water requires agents that treat Pseudomonas species, such as ceftazidime or cefepime. Fluoroquinolones are not indicated as monotherapy for patients with type III open fractures and may have a detrimental effect on fracture healing. The duration of antibiotic prophylaxis varies based on fracture type, with type I and II open fractures requiring antibiotic treatment for 24 hours while type III fractures should be prophylactically treated for 72 hours or 24 hours after soft tissue injuries have been closed. Open fractures are also more susceptible to tetanus, and the immunization status of all patients should be determined with appropriate administration of tetanus toxoid, diphtheria-tetanus-acellular pertussis, booster tetanus toxoid-reduced diphtheria toxoid-acellular pertussis, or tetanus-diphtheria toxoids adsorbed.
What is the bursal fluid white blood cell count that distinguishes inflammation from infection?
2,000 white blood cells per mm3.
When should a bone mineral density scan be repeated in an untreated patient with a T-score of -1.35? A 10 years B One year C Six months D Three years
A 10 years Osteopenia is a term used to describe low bone mineral mass. Although not a disease itself, patients with osteopenia have an increased risk of fracture due to disruption of the cortical and trabecular bone matrices. Bone mineral mass decreases proportionally with advanced age and is most common in postmenopausal women. Independent risk factors for fracture secondary to decreased bone mineral density include advancing age, prior fracture, glucocorticoid therapy, parental history of hip fracture, low body weight, current cigarette smoking, excessive alcohol consumption, rheumatoid arthritis, and secondary osteoporosis (e.g., untreated hyperthyroidism, inflammatory bowel disease, hypogonadism, premature menopause). Bone mineral density is described in the difference of standard deviations between a patient and a young adult reference as a T-score. Osteoporosis is indicated with a value ≤ -2.5, with osteopenia values ranging between -1.0 and -2.5, and normal bone density values ≥ -1.0. Osteopenia is generally asymptomatic until a fracture occurs. Vertebral fractures may present with backache with loss of height. Other common fracture sites include the hip, pelvis, wrist, and proximal humerus. The presence of osteoporosis is not required for patients to sustain a fracture due to low bone mineral density. The Fracture Risk Assessment Tool can be used to calculate a 10-year probability of hip or major osteoporotic fracture in untreated patients. Plain film radiography cannot diagnose low bone mineral density but may be used to evaluate patients for acute fractures. Monitoring of bone mineral density is accomplished using dual-energy X-ray absorptiometry (DEXA). Testing is most commonly performed at the spine and hip, but forearm testing can be used in patients with prior fractures to the hip or spine. In the osteopenic patient population, those with a T-score between -1.50 to -1.99 should be retested every three to five years while those with scores between -1.01 to -1.49 can be monitored every 10 to 15 years. Patients with T-scores between -2.00 to -2.49 or who have ongoing risk factors for osteoporosis (e.g., hyperparathyroidism, glucocorticoid use) should be monitored for progression of disease every two years. Postmenopausal women and men 50 years of age or older with a T-score between -1.0 and -2.5 and a 10-year probability of hip fracture ≥ 3% or any major osteoporotic fracture ≥ 20% should be treated. Initial treatment involves oral bisphosphonates (e.g., alendronate, risedronate), which are administered once per week or intravenous preparations, such as zoledronate, which are administered annually. Parathyroid hormone derivatives (e.g., teriparatide), monoclonal antibody medications (e.g., denosumab), and selective estrogen receptor modulators (e.g., raloxifene, tamoxifen) are alternative therapies for patients who cannot tolerate bisphosphonates. A "drug holiday" is required every five years for low-risk patients wherein the medication is held for three to five years or until two concurrent dual-energy X-ray absorptiometry scans at least two years apart show bone loss of greater than 5%. Patients with high fracture risk can continue bisphosphonate treatment for 10 years prior to drug cessation. Education and risk factor reduction is essential to slowing the progression of bone mineral loss. All patients should take daily calcium 1,200 mg orally and vitamin D 600-1,000 IU orally. Patients on medical regimens should undergo repeat dual-energy X-ray absorptiometry scan one to two years after initiating or changing treatment.
A 30-year-old man presents to the emergency department with pleuritic chest pain that worsens when taking deep breaths and lying on his back but is better with sitting and leaning forward. Physical exam reveals a pericardial friction rub. Which of the following electrocardiogram results will be classically seen in a patient with this condition? A Electrical alternans B Peaked T waves C ST elevation in leads V2 or V3 D Widespread ST elevation in multiple leads
D Widespread ST elevation in multiple leads Acute pericarditis occurs when there is inflammation of the pericardium. There are multiple etiologies of this condition, including idiopathic causes, infectious causes (e.g., viral, bacterial, tuberculous infections), inflammatory disorders (e.g., rheumatoid arthritis, systemic lupus erythematosus, scleroderma, rheumatic fever), metabolic disorders (e.g., renal failure, hypothyroidism), cardiovascular disorders (e.g., acute myocardial infarction, Dressler syndrome, aortic dissection), and other causes (e.g., neoplasms, iatrogenic cardiac procedures, trauma). Although there are many etiologies, viral infection is the most common cause of this condition, with some of the most common viruses being coxsackievirus B, echovirus, adenovirus, influenza A and B viruses, and enterovirus. Patients will present with chest pain (cardinal symptom) that is precordial or retrosternal and will refer to the trapezius, neck, left shoulder, or arm. The pain is pleuritic in quality but can also be described as sharp, dull, aching, burning, and pressing. Pain is worse during inspiration, when lying flat, during swallowing, and with certain motions. It is relieved with sitting and leaning forward. Other signs and symptoms include low-grade fever, dyspnea, tachypnea, cough, and dysphagia. There will be a pericardial friction rub (pathognomonic for acute pericarditis) on auscultation best heard with the diaphragm of the stethoscope. The rub will also change in character with position changes. An electrocardiogram (ECG) will reveal classic widespread ST elevation or PR depression, indicating a new or worsening pericardial effusion. Diagnosis is based on clinical history, physical exam findings, ECG findings, echocardiography, chest radiography, and laboratory studies. An echocardiogram is indicated if there is suspicion of pericardial effusion, myocarditis, or purulent pericarditis. A chest radiograph should be ordered to exclude other associated causes of pericarditis, but will not give any evidence of the presence of pericardial fluid. A complete blood count, serum electrolyte, blood urea nitrogen, creatinine, erythrocyte sedimentation rate, C-reactive protein, cardiac biomarkers, and lactate dehydrogenase should be ordered as well to rule out underlying causes of pericarditis. Treatment is directed according to the underlying cause. Patients with idiopathic and viral pericarditis are treated symptomatically with aspirin and nonsteroidal anti-inflammatory drugs (NSAIDs) as the mainstay of treatment. Colchicine can also be used as first-line therapy in combination with aspirin or an NSAID. Corticosteroids should not be used for initial treatment, but only if there is underlying disease that warrants its usage or if the patient does not respond to NSAIDs or colchicine. Surgical procedures such as pericardiectomy (most effective for managing large effusions), pericardiocentesis (for effusions with evidence of tamponade), pericardial window placement, and pericardiotomy may also be indicated in severe cases.
A 40-year-old woman presents to the emergency department with worsening dyspnea during exercise. She has a history of rheumatic heart disease. Which of the following physical exam findings is most likely to be present? AA harsh crescendo-decrescendo systolic murmur BA holosystolic murmur heard over the apex of the heart radiating to the back CA loud S1 heart sound and a normal S2 sound, followed by an opening snap and a low-pitched diastolic rumbling murmur DA midsystolic click followed by a high-pitched late systolic murmur at the cardiac apex
C A loud S1 heart sound and a normal S2 sound, followed by an opening snap and a low-pitched diastolic rumbling murmur Mitral stenosis is characterized by stenosis of the mitral valve leading to increased pressure across the valve, resulting in increased left atrial pressure, accumulation of fluid in the lungs, right ventricular dilation, and pulmonary hypertension. It has multiple etiologies, including rheumatic heart disease (most common cause), congenital structural abnormalities, and mitral valve calcification (more common in the elderly). The normal diameter of the mitral valve is around 4-6 cm2 and as the valve becomes more stenosed, the pressure across the mitral valve increases to ensure adequate flow. Women are more likely than men to develop rheumatic mitral stenosis, and the onset of symptoms occurs between the third and fourth decade of life. In mild-to-moderate cases of mitral stenosis, patients are asymptomatic early on, but those who are symptomatic present with progressive exertional dyspnea that may be triggered by any event that causes the heart rate to be elevated (e.g., fever, severe anemia, exercise, pregnancy). In severe cases, marked pulmonary hypertension develops, causing symptoms of right-sided heart failure (e.g., pulmonary edema, shortness of breath, swelling of the lower extremities) and low cardiac output. Most patients will eventually develop atrial fibrillation due to the development of left atrial dilatation and therefore increases the risk of thromboembolism. Characteristic physical exam findings include a loud S1, with an opening snap following a normal S2 heart sound (due to the stiff mitral valve). A low-pitched diastolic rumbling murmur will be present, best heard with the bell of the stethoscope at the apex of the heart when the patient is in the left lateral decubitus position. Patients with severe disease may also present with a malar flush (pinkish-purplish patches on the cheeks), jugular venous distension due to pulmonary hypertension, and right-sided heart failure. Echocardiography is the tool of choice to diagnose and assess the severity of the disease. It is the most specific and sensitive method due to its ability to note anatomic abnormalities (e.g. presence of calcification of the valve as well as its mobility and motion) and can also give information about the atrial and ventricular size, any presence of thrombi, and determine if any invasive procedures may be required (e.g., cardiac catheterization). Percutaneous balloon valvuloplasty is preferred over surgical valve replacement in patients with symptoms or evidence of pulmonary hypertension or pulmonary edema. No intervention is required in asymptomatic patients.
At what level of arterial carbon dioxide tension do patients with chronic hypercapnia begin to exhibit an altered level of consciousness?
PaCO2 greater than 90-100 mm Hg
Which is a urologic emergency: phimosis or paraphimosis?
Paraphimosis is the trapping of the prepuce behind the glans penis causing a limited lymphatic and venous outflow while allowing continued arterial flow.
What are the indications for surgical repair of a Bankart lesion?
Patients with recurrent dislocations and certain young patients who want to return to high-risk activities.
What class of medications is indicated to additionally lower low-density lipoprotein levels in patients with maximally tolerated statin therapy?
Proprotein convertase subtilisin-kexin type 9 (PCSK9) inhibitors, such as evolocumab and alirocumab.
What drug class is associated with precipitating a manic episode in patients with bipolar I or II disorder?
Selective serotonin reuptake inhibitors.
What is the goal level with uric acid maintenance therapy?
Serum uric acid at or below 6 mg/dL.
What is the role of melatonin in hepatic encephalopathy?
Sleep disturbance is a common symptom of hepatic encephalopathy and may be corrected with use of melatonin, but studies have shown no significant clinical improvement with its use.